Sie sind auf Seite 1von 52

THIRD DIVISION

[G.R. No. 146006. February 23, 2004]

JOSE C. LEE AND ALMA AGGABAO, in their capacities as President and Corporate Secretary,
respectively, of Philippines Internationl Life Insurance Company, and FILIPINO LOAN
ASSISTANCE GROUP, petitioners, vs. REGIONAL TRIAL COURT OF QUEZON CITY
BRANCH 85 presided by JUDGE PEDRO M. AREOLA, BRANCH CLERK OF COURT
JANICE Y. ANTERO, DEPUTY SHERIFFS ADENAUER G. RIVERA and PEDRO L.
BORJA, all of the Regional Trial Court of Quezon City Branch 85, MA. DIVINA
ENDERES claiming to be Special Administratrix, and other persons/ public officers acting
for and in their behalf, respondents.
DECISION
CORONA, J.:
This is a petition for review under Rule 45 of the Rules of Court seeking to reverse and set aside the
decision[1] of the Court of Appeals, First Division, dated July 26, 2000, in CA G.R. 59736, which
dismissed the petition for certiorari filed by petitioners Jose C. Lee and Alma Aggabao (in their capacities
as president and secretary, respectively, of Philippine International Life Insurance Company) and Filipino
Loan Assistance Group.
The antecedent facts follow.
Dr. Juvencio P. Ortaez incorporated the Philippine International Life Insurance Company, Inc. on
July 6, 1956. At the time of the companys incorporation, Dr. Ortaez owned ninety percent (90%) of the
subscribed capital stock.
On July 21, 1980, Dr. Ortaez died. He left behind a wife (Juliana Salgado Ortaez), three legitimate
children (Rafael, Jose and Antonio Ortaez) and five illegitimate children by Ligaya Novicio (herein
private respondent Ma. Divina Ortaez-Enderes and her siblings Jose, Romeo, Enrico Manuel and Cesar,
all surnamed Ortaez).[2]
On September 24, 1980, Rafael Ortaez filed before the Court of First Instance of Rizal, Quezon City
Branch (now Regional Trial Court of Quezon City) a petition for letters of administration of the intestate
estate of Dr. Ortaez, docketed as SP Proc. Q-30884 (which petition to date remains pending at Branch 85
thereof).
Private respondent Ma. Divina Ortaez-Enderes and her siblings filed an opposition to the petition for
letters of administration and, in a subsequent urgent motion, prayed that the intestate court appoint a
special administrator.
On March 10, 1982, Judge Ernani Cruz Pao, then presiding judge of Branch 85, appointed Rafael
and Jose Ortaez joint special administrators of their fathers estate. Hearings continued for the appointment
of a regular administrator (up to now no regular administrator has been appointed).
As ordered by the intestate court, special administrators Rafael and Jose Ortaez submitted an
inventory of the estate of their father which included, among other properties, 2,029[3]shares of stock in

Philippine International Life Insurance Company (hereafter Philinterlife), representing 50.725% of the
companys outstanding capital stock.
On April 15, 1989, the decedents wife, Juliana S. Ortaez, claiming that she owned
1,014[4] Philinterlife shares of stock as her conjugal share in the estate, sold said shares with right to
repurchase in favor of herein petitioner Filipino Loan Assistance Group (FLAG), represented by its
president, herein petitioner Jose C. Lee. Juliana Ortaez failed to repurchase the shares of stock within the
stipulated period, thus ownership thereof was consolidated by petitioner FLAG in its name.
On October 30, 1991, Special Administrator Jose Ortaez, acting in his personal capacity and
claiming that he owned the remaining 1,011[5] Philinterlife shares of stocks as his inheritance share in the
estate, sold said shares with right to repurchase also in favor of herein petitioner FLAG, represented by its
president, herein petitioner Jose C. Lee. After one year, petitioner FLAG consolidated in its name the
ownership of the Philinterlife shares of stock when Jose Ortaez failed to repurchase the same.
It appears that several years before (but already during the pendency of the intestate proceedings at
the Regional Trial Court of Quezon City, Branch 85), Juliana Ortaez and her two children, Special
Administrators Rafael and Jose Ortaez, entered into a memorandum of agreement dated March 4, 1982
for the extrajudicial settlement of the estate of Dr. Juvencio Ortaez, partitioning the estate (including the
Philinterlife shares of stock) among themselves. This was the basis of the number of shares separately
sold by Juliana Ortaez on April 15, 1989 (1,014 shares) and by Jose Ortaez on October 30, 1991 (1,011
shares) in favor of herein petitioner FLAG.
On July 12, 1995, herein private respondent Ma. Divina OrtaezEnderes and her siblings (hereafter
referred to as private respondents Enderes et al.) filed a motion for appointment of special administrator
of Philinterlife shares of stock. This move was opposed by Special Administrator Jose Ortaez.
On November 8, 1995, the intestate court granted the motion of private respondents Enderes et
al. and appointed private respondent Enderes special administratrix of the Philinterlife shares of stock.
On December 20, 1995, Special Administratrix Enderes filed an urgent motion to declare void ab
initio the memorandum of agreement dated March 4, 1982. On January 9, 1996, she filed a motion to
declare the partial nullity of the extrajudicial settlement of the decedents estate. These motions were
opposed by Special Administrator Jose Ortaez.
On March 22, 1996, Special Administratrix Enderes filed an urgent motion to declare void ab
initio the deeds of sale of Philinterlife shares of stock, which move was again opposed by Special
Administrator Jose Ortaez.
On February 4, 1997, Jose Ortaez filed an omnibus motion for (1) the approval of the deeds of sale
of the Philinterlife shares of stock and (2) the release of Ma. Divina Ortaez-Enderes as special
administratrix of the Philinterlife shares of stock on the ground that there were no longer any shares of
stock for her to administer.
On August 11, 1997, the intestate court denied the omnibus motion of Special Administrator Jose
Ortaez for the approval of the deeds of sale for the reason that:
Under the Godoy case, supra, it was held in substance that a sale of a property of the estate without an
Order of the probate court is void and passes no title to the purchaser. Since the sales in question were
entered into by Juliana S. Ortaez and Jose S. Ortaez in their personal capacity without prior approval of
the Court, the same is not binding upon the Estate.
WHEREFORE, the OMNIBUS MOTION for the approval of the sale of Philinterlife shares of stock and
release of Ma. Divina Ortaez-Enderes as Special Administratrix is hereby denied.[6]

On August 29, 1997, the intestate court issued another order granting the motion of Special
Administratrix Enderes for the annulment of the March 4, 1982 memorandum of agreement or
extrajudicial partition of estate. The court reasoned that:
In consonance with the Order of this Court dated August 11, 1997 DENYING the approval of the sale of
Philinterlife shares of stocks and release of Ma. Divina Ortaez-Enderes as Special Administratrix, the
Urgent Motion to Declare Void Ab Initio Memorandum of Agreement dated December 19, 1995. . . is
hereby impliedly partially resolved insofar as the transfer/waiver/renunciation of the Philinterlife shares
of stock are concerned, in particular, No. 5, 9(c), 10(b) and 11(d)(ii) of the Memorandum of Agreement.
WHEREFORE, this Court hereby declares the Memorandum of Agreement dated March 4, 1982
executed by Juliana S. Ortaez, Rafael S. Ortaez and Jose S. Ortaez as partially void ab initio insofar as the
transfer/waiver/renunciation of the Philinterlife shares of stocks are concerned.[7]
Aggrieved by the above-stated orders of the intestate court, Jose Ortaez filed, on December 22, 1997,
a petition for certiorari in the Court of Appeals. The appellate court denied his petition, however, ruling
that there was no legal justification whatsoever for the extrajudicial partition of the estate by Jose Ortaez,
his brother Rafael Ortaez and mother Juliana Ortaez during the pendency of the settlement of the estate of
Dr. Ortaez, without the requisite approval of the intestate court, when it was clear that there were other
heirs to the estate who stood to be prejudiced thereby. Consequently, the sale made by Jose Ortaez and his
mother Juliana Ortaez to FLAG of the shares of stock they invalidly appropriated for themselves, without
approval of the intestate court, was void.[8]
Special Administrator Jose Ortaez filed a motion for reconsideration of the Court of Appeals
decision but it was denied. He elevated the case to the Supreme Court via petition for review under Rule
45 which the Supreme Court dismissed on October 5, 1998, on a technicality. His motion for
reconsideration was denied with finality on January 13, 1999. On February 23, 1999, the resolution of the
Supreme Court dismissing the petition of Special Administrator Jose Ortaez became final and was
subsequently recorded in the book of entries of judgments.
Meanwhile, herein petitioners Jose Lee and Alma Aggabao, with the rest of the FLAG-controlled
board of directors, increased the authorized capital stock of Philinterlife, diluting in the process the
50.725% controlling interest of the decedent, Dr. Juvencio Ortaez, in the insurance company. [9] This
became the subject of a separate action at the Securities and Exchange Commission filed by private
respondent-Special Administratrix Enderes against petitioner Jose Lee and other members of the FLAGcontrolled board of Philinterlife on November 7, 1994. Thereafter, various cases were filed by Jose Lee as
president of Philinterlife and Juliana Ortaez and her sons against private respondent-Special
Administratrix Enderes in the SEC and civil courts.[10] Somehow, all these cases were connected to the
core dispute on the legality of the sale of decedent Dr. Ortaezs Philinterlife shares of stock to petitioner
FLAG, represented by its president, herein petitioner Jose Lee who later became the president of
Philinterlife after the controversial sale.
On May 2, 2000, private respondent-Special Administratrix Enderes and her siblings filed a motion
for execution of the Orders of the intestate court dated August 11 and August 29, 1997 because the orders
of the intestate court nullifying the sale (upheld by the Court of Appeals and the Supreme Court) had long
became final. Respondent-Special Administratrix Enderes served a copy of the motion to petitioners Jose
Lee and Alma Aggabao as president and secretary, respectively, of Philinterlife,[11] but petitioners ignored
the same.
On July 6, 2000, the intestate court granted the motion for execution, the dispositive portion of which
read:

WHEREFORE, premises considered, let a writ of execution issue as follows:


1. Confirming the nullity of the sale of the 2,029 Philinterlife shares in the name of the
Estate of Dr. Juvencio Ortaez to Filipino Loan Assistance Group (FLAG);
2. Commanding the President and the Corporate Secretary of Philinterlife to reinstate in the
stock and transfer book of Philinterlife the 2,029 Philinterlife shares of stock in the
name of the Estate of Dr. Juvencio P. Ortaez as the owner thereof without prejudice
to other claims for violation of pre-emptive rights pertaining to the said 2,029
Philinterlife shares;
3. Directing the President and the Corporate Secretary of Philinterlife to issue stock
certificates of Philinterlife for 2,029 shares in the name of the Estate of Dr. Juvencio
P. Ortaez as the owner thereof without prejudice to other claims for violations of
pre-emptive rights pertaining to the said 2,029 Philinterlife shares and,
4. Confirming that only the Special Administratrix, Ma. Divina Ortaez-Enderes, has the
power to exercise all the rights appurtenant to the said shares, including the right to
vote and to receive dividends.
5. Directing Philinterlife and/or any other person or persons claiming to represent it or
otherwise, to acknowledge and allow the said Special Administratrix to exercise all
the aforesaid rights on the said shares and to refrain from resorting to any action
which may tend directly or indirectly to impede, obstruct or bar the free exercise
thereof under pain of contempt.
6. The President, Corporate Secretary, any responsible officer/s of Philinterlife, or any other
person or persons claiming to represent it or otherwise, are hereby directed to
comply with this order within three (3) days from receipt hereof under pain of
contempt.
7. The Deputy Sheriffs Adenauer Rivera and Pedro Borja are hereby directed to implement
the writ of execution with dispatch to forestall any and/or further damage to the
Estate.
SO ORDERED.[12]
In the several occasions that the sheriff went to the office of petitioners to execute the writ of
execution, he was barred by the security guard upon petitioners instructions. Thus, private respondentSpecial Administratrix Enderes filed a motion to cite herein petitioners Jose Lee and Alma Aggabao
(president and secretary, respectively, of Philinterlife) in contempt.[13]
Petitioners Lee and Aggabao subsequently filed before the Court of Appeals a petition for certiorari,
docketed as CA G.R. SP No. 59736. Petitioners alleged that the intestate court gravely abused its
discretion in (1) declaring that the ownership of FLAG over the Philinterlife shares of stock was null and
void; (2) ordering the execution of its order declaring such nullity and (3) depriving the petitioners of
their right to due process.
On July 26, 2000, the Court of Appeals dismissed the petition outright:
We are constrained to DISMISS OUTRIGHT the present petition for certiorari and prohibition with
prayer for a temporary restraining order and/or writ of preliminary injunction in the light of the following
considerations:

1. The assailed Order dated August 11, 1997 of the respondent judge had long become final
and executory;
2. The certification on non-forum shopping is signed by only one (1) of the three (3)
petitioners in violation of the Rules; and
3. Except for the assailed orders and writ of execution, deed of sale with right to repurchase,
deed of sale of shares of stocks and omnibus motion, the petition is not accompanied
by such pleadings, documents and other material portions of the record as would
support the allegations therein in violation of the second paragraph, Rule 65 of the
1997 Rules of Civil Procedure, as amended.
Petition is DISMISSED.
SO ORDERED.[14]
The motion for reconsideration filed by petitioners Lee and Aggabao of the above decision was
denied by the Court of Appeals on October 30, 2000:
This resolves the urgent motion for reconsideration filed by the petitioners of our resolution of July 26,
2000 dismissing outrightly the above-entitled petition for the reason, among others, that the assailed
Order dated August 11, 1997 of the respondent Judge had long become final and executory.
Dura lex, sed lex.
WHEREFORE, the urgent motion for reconsideration is hereby DENIED, for lack of merit.
SO ORDERED.[15]
On December 4, 2000, petitioners elevated the case to the Supreme Court through a petition for
review under Rule 45 but on December 13, 2000, we denied the petition because there was no showing
that the Court of Appeals in CA G.R. SP No. 59736 committed any reversible error to warrant the
exercise by the Supreme Court of its discretionary appellate jurisdiction.[16]
However, upon motion for reconsideration filed by petitioners Lee and Aggabao, the Supreme Court
granted the motion and reinstated their petition on September 5, 2001. The parties were then required to
submit their respective memoranda.
Meanwhile, private respondent-Special Administratrix Enderes, on July 19, 2000, filed a motion to
direct the branch clerk of court in lieu of herein petitioners Lee and Aggabao to reinstate the name of Dr.
Ortaez in the stock and transfer book of Philinterlife and issue the corresponding stock certificate
pursuant to Section 10, Rule 39 of the Rules of Court which provides that the court may direct the act to
be done at the cost of the disobedient party by some other person appointed by the court and the act when
so done shall have the effect as if done by the party. Petitioners Lee and Aggabao opposed the motion on
the ground that the intestate court should refrain from acting on the motion because the issues raised
therein were directly related to the issues raised by them in their petition for certiorari at the Court of
Appeals docketed as CA-G.R. SP No. 59736. On October 30, 2000, the intestate court granted the motion,
ruling that there was no prohibition for the intestate court to execute its orders inasmuch as the appellate
court did not issue any TRO or writ of preliminary injunction.
On December 3, 2000, petitioners Lee and Aggabao filed a petition for certiorari in the Court of
Appeals, docketed as CA-G.R. SP No. 62461, questioning this time the October 30, 2000 order of the

intestate court directing the branch clerk of court to issue the stock certificates. They also questioned in
the Court of Appeals the order of the intestate court nullifying the sale made in their favor by Juliana
Ortaez and Jose Ortaez. On November 20, 2002, the Court of Appeals denied their petition and upheld the
power of the intestate court to execute its order. Petitioners Lee and Aggabao then filed motion for
reconsideration which at present is still pending resolution by the Court of Appeals.
Petitioners Jose Lee and Alma Aggabao (president and secretary, respectively, of Philinterlife) and
FLAG now raise the following errors for our consideration:
THE COURT OF APPEALS COMMITTED GRAVE REVERSIBLE ERROR:
A. IN FAILING TO RECONSIDER ITS PREVIOUS RESOLUTION DENYING THE
PETITION DESPITE THE FACT THAT THE APPELLATE COURTS MISTAKE IN
APPREHENDING THE FACTS HAD BECOME PATENT AND EVIDENT FROM
THE MOTION FOR RECONSIDERATION AND THE COMMENT OF
RESPONDENT ENDERES WHICH HAD ADMITTED THE FACTUAL
ALLEGATIONS OF PETITIONERS IN THE PETITION AS WELL AS IN THE
MOTION FOR RECONSIDERATION. MOREOVER, THE RESOLUTION OF THE
APPELLATE COURT DENYING THE MOTION FOR RECONSIDERATION WAS
CONTAINED IN ONLY ONE PAGE WITHOUT EVEN TOUCHING ON THE
SUBSTANTIVE MERITS OF THE EXHAUSTIVE DISCUSSION OF FACTS AND
SUPPORTING LAW IN THE MOTION FOR RECONSIDERATION IN
VIOLATION OF THE RULE ON ADMINISTRATIVE DUE PROCESS;
B. IN FAILING TO SET ASIDE THE VOID ORDERS OF THE INTESTATE COURT
ON THE ERRONEOUS GROUND THAT THE ORDERS WERE FINAL AND
EXECUTORY WITH REGARD TO PETITIONERS EVEN AS THE LATTER
WERE NEVER NOTIFIED OF THE PROCEEDINGS OR ORDER CANCELING
ITS OWNERSHIP;
C. IN NOT FINDING THAT THE INTESTATE COURT COMMITTED GRAVE
ABUSE OF DISCRETION AMOUNTING TO EXCESS OF JURISDICTION (1)
WHEN IT ISSUED THE OMNIBUS ORDER NULLIFYING THE OWNERSHIP OF
PETITIONER FLAG OVER SHARES OF STOCK WHICH WERE ALLEGED TO
BE PART OF THE ESTATE AND (2) WHEN IT ISSUED A VOID WRIT OF
EXECUTION AGAINST PETITIONER FLAG AS PRESENT OWNER TO
IMPLEMENT MERELY PROVISIONAL ORDERS, THEREBY VIOLATING
FLAGS CONSTITUTIONAL RIGHT AGAINST DEPRIVATION OF PROPERTY
WITHOUT DUE PROCESS;
D. IN FAILING TO DECLARE NULL AND VOID THE ORDERS OF THE
INTESTATE COURT WHICH NULLIFIED THE SALE OF SHARES OF STOCK
BETWEEN THE LEGITIMATE HEIR JOSE S. ORTAEZ AND PETITIONER FLAG
BECAUSE OF SETTLED LAW AND JURISPRUDENCE, I.E., THAT AN HEIR
HAS THE RIGHT TO DISPOSE OF THE DECEDENTS PROPERTY EVEN IF THE
SAME IS UNDER ADMINISTRATION PURSUANT TO CIVIL CODE PROVISION
THAT POSSESSION OF HEREDITARY PROPERTY IS TRANSMITTED TO THE
HEIR THE MOMENT OF DEATH OF THE DECEDENT (ACEDEBO VS.
ABESAMIS, 217 SCRA 194);

E. IN DISREGARDING THE FINAL DECISION OF THE SUPREME COURT IN G.R.


NO. 128525 DATED DECEMBER 17, 1999 INVOLVING SUBSTANTIALLY THE
SAME PARTIES, TO WIT, PETITIONERS JOSE C. LEE AND ALMA AGGABAO
WERE RESPONDENTS IN THAT CASE WHILE RESPONDENT MA. DIVINA
ENDERES WAS THE PETITIONER THEREIN. THAT DECISION, WHICH CAN
BE CONSIDERED LAW OF THE CASE, RULED THAT PETITIONERS CANNOT
BE ENJOINED BY RESPONDENT ENDERES FROM EXERCISING THEIR
POWER AS DIRECTORS AND OFFICERS OF PHILINTERLIFE AND THAT THE
INTESTATE COURT IN CHARGE OF THE INTESTATE PROCEEDINGS
CANNOT ADJUDICATE TITLE TO PROPERTIES CLAIMED TO BE PART OF
THE ESTATE AND WHICH ARE EQUALLY CLAIMED BY PETITIONER
FLAG.[17]
The petition has no merit.
Petitioners Jose Lee and Alma Aggabao, representing Philinterlife and FLAG, assail before us not
only the validity of the writ of execution issued by the intestate court dated July 7, 2000 but also the
validity of the August 11, 1997 order of the intestate court nullifying the sale of the 2,029 Philinterlife
shares of stock made by Juliana Ortaez and Jose Ortaez, in their personal capacities and without court
approval, in favor of petitioner FLAG.
We cannot allow petitioners to reopen the issue of nullity of the sale of the Philinterlife shares of
stock in their favor because this was already settled a long time ago by the Court of Appeals in its
decision dated June 23, 1998 in CA-G.R. SP No. 46342. This decision was effectively upheld by us in our
resolution dated October 9, 1998 in G.R. No. 135177 dismissing the petition for review on a technicality
and thereafter denying the motion for reconsideration on January 13, 1999 on the ground that there was
no compelling reason to reconsider said denial.[18] Our decision became final on February 23, 1999 and
was accordingly entered in the book of entry of judgments. For all intents and purposes therefore, the
nullity of the sale of the Philinterlife shares of stock made by Juliana Ortaez and Jose Ortaez in favor of
petitioner FLAG is already a closed case. To reopen said issue would set a bad precedent, opening the
door wide open for dissatisfied parties to relitigate unfavorable decisions no end. This is completely
inimical to the orderly and efficient administration of justice.
The said decision of the Court of Appeals in CA-G.R. SP No. 46342 affirming the nullity of the sale
made by Jose Ortaez and his mother Juliana Ortaez of the Philinterlife shares of stock read:
Petitioners asseverations relative to said [memorandum] agreement were scuttled during the hearing
before this Court thus:
JUSTICE AQUINO:
Counsel for petitioner, when the Memorandum of Agreement was executed, did the
children of Juliana Salgado know already that there was a claim for share in the inheritance
of the children of Novicio?
ATTY. CALIMAG:
Your Honor please, at that time, Your Honor, it is already known to them.
JUSTICE AQUINO:
What can be your legal justification for extrajudicial settlement of a property subject
of intestate proceedings when there is an adverse claim of another set of heirs, alleged

heirs? What would be the legal justification for extra-judicially settling a property under
administration without the approval of the intestate court?
ATTY. CALIMAG:
Well, Your Honor please, in that extra-judicial settlement there is an approval of the
honorable court as to the propertys partition x x x. There were as mentioned by the
respondents counsel, Your Honor.
ATTY. BUYCO:
No
JUSTICE AQUINO:
The point is, there can be no adjudication of a property under intestate proceedings
without the approval of the court. That is basic unless you can present justification on that.
In fact, there are two steps: first, you ask leave and then execute the document and then ask
for approval of the document executed. Now, is there any legal justification to exclude this
particular transaction from those steps?
ATTY. CALIMAG:
None, Your Honor.
ATTY BUYCO:
With that admission that there is no legal justification, Your Honor, we rest the case
for the private respondent. How can the lower court be accused of abusing its discretion?
(pages 33-35, TSN of January 29, 1998).
Thus, We find merit in the following postulation by private respondent:
What we have here is a situation where some of the heirs of the decedent without securing court approval
have appropriated as their own personal property the properties of [the] Estate, to the exclusion and the
extreme prejudice of the other claimant/heirs. In other words, these heirs, without court approval, have
distributed the asset of the estate among themselves and proceeded to dispose the same to third parties
even in the absence of an order of distribution by the Estate Court. As admitted by petitioners counsel,
there was absolutely no legal justification for this action by the heirs. There being no legal justification,
petitioner has no basis for demanding that public respondent [the intestate court] approve the sale of the
Philinterlife shares of the Estate by Juliana and Jose Ortaez in favor of the Filipino Loan Assistance
Group.
It is an undisputed fact that the parties to the Memorandum of Agreement dated March 4, 1982 (see
Annex 7 of the Comment). . . are not the only heirs claiming an interest in the estate left by Dr. Juvencio
P. Ortaez. The records of this case. . . clearly show that as early as March 3, 1981 an Opposition to the
Application for Issuance of Letters of Administration was filed by the acknowledged natural children of
Dr. Juvencio P. Ortaez with Ligaya Novicio. . . This claim by the acknowledged natural children of Dr.
Juvencio P. Ortaez is admittedly known to the parties to the Memorandum of Agreement before they
executed the same. This much was admitted by petitioners counsel during the oral argument. xxx
Given the foregoing facts, and the applicable jurisprudence, public respondent can never be faulted for
not approving. . . the subsequent sale by the petitioner [Jose Ortaez] and his mother [Juliana Ortaez] of
the Philinterlife shares belonging to the Estate of Dr. Juvencio P. Ortaez. (pages 3-4 of Private
Respondents Memorandum; pages 243-244 of the Rollo)

Amidst the foregoing, We found no grave abuse of discretion amounting to excess or want of jurisdiction
committed by respondent judge.[19]
From the above decision, it is clear that Juliana Ortaez, and her three sons, Jose, Rafael and Antonio,
all surnamed Ortaez, invalidly entered into a memorandum of agreement extrajudicially partitioning the
intestate estate among themselves, despite their knowledge that there were other heirs or claimants to the
estate and before final settlement of the estate by the intestate court. Since the appropriation of the estate
properties by Juliana Ortaez and her children (Jose, Rafael and Antonio Ortaez) was invalid, the
subsequent sale thereof by Juliana and Jose to a third party (FLAG), without court approval, was likewise
void.
An heir can sell his right, interest, or participation in the property under administration under Art.
533 of the Civil Code which provides that possession of hereditary property is deemed transmitted to the
heir without interruption from the moment of death of the decedent.[20] However, an heir can only alienate
such portion of the estate that may be allotted to him in the division of the estate by the probate or
intestate court after final adjudication, that is, after all debtors shall have been paid or the devisees or
legatees shall have been given their shares.[21]This means that an heir may only sell his ideal or undivided
share in the estate, not any specific property therein. In the present case, Juliana Ortaez and Jose Ortaez
sold specific properties of the estate (1,014 and 1,011 shares of stock in Philinterlife) in favor of petitioner
FLAG. This they could not lawfully do pending the final adjudication of the estate by the intestate court
because of the undue prejudice it would cause the other claimants to the estate, as what happened in the
present case.
Juliana Ortaez and Jose Ortaez sold specific properties of the estate, without court approval. It is
well-settled that court approval is necessary for the validity of any disposition of the decedents estate. In
the early case of Godoy vs. Orellano,[22] we laid down the rule that the sale of the property of the estate by
an administrator without the order of the probate court is void and passes no title to the purchaser. And in
the case of Dillena vs. Court of Appeals,[23] we ruled that:
[I]t must be emphasized that the questioned properties (fishpond) were included in the inventory of
properties of the estate submitted by then Administratrix Fausta Carreon Herrera on November 14, 1974.
Private respondent was appointed as administratrix of the estate on March 3, 1976 in lieu of Fausta
Carreon Herrera. On November 1, 1978, the questioned deed of sale of the fishponds was executed
between petitioner and private respondent without notice and approval of the probate court. Even after the
sale, administratrix Aurora Carreon still included the three fishponds as among the real properties of the
estate in her inventory submitted on August 13, 1981. In fact, as stated by the Court of Appeals,
petitioner, at the time of the sale of the fishponds in question, knew that the same were part of the estate
under administration.
xxxxxxxxx
The subject properties therefore are under the jurisdiction of the probate court which according to our
settled jurisprudence has the authority to approve any disposition regarding properties under
administration. . . More emphatic is the declaration We made in Estate of Olave vs. Reyes (123 SCRA
767) where We stated that when the estate of the deceased person is already the subject of a testate or
intestate proceeding, the administrator cannot enter into any transaction involving it without prior
approval of the probate court.
Only recently, in Manotok Realty, Inc. vs. Court of Appeals (149 SCRA 174), We held that the sale of an
immovable property belonging to the estate of a decedent, in a special proceedings, needs court approval.
. . This pronouncement finds support in the previous case of Dolores Vda. De Gil vs. Agustin Cancio (14

SCRA 797) wherein We emphasized that it is within the jurisdiction of a probate court to approve the sale
of properties of a deceased person by his prospective heirs before final adjudication. x x x
It being settled that property under administration needs the approval of the probate court before it can be
disposed of, any unauthorized disposition does not bind the estate and is null and void. As early as 1921
in the case of Godoy vs. Orellano (42 Phil 347), We laid down the rule that a sale by an administrator of
property of the deceased, which is not authorized by the probate court is null and void and title does not
pass to the purchaser.
There is hardly any doubt that the probate court can declare null and void the disposition of the property
under administration, made by private respondent, the same having been effected without authority from
said court. It is the probate court that has the power to authorize and/or approve the sale (Section 4 and
7, Rule 89), hence, a fortiori, it is said court that can declare it null and void for as long as the
proceedings had not been closed or terminated. To uphold petitioners contention that the probate court
cannot annul the unauthorized sale, would render meaningless the power pertaining to the said court.
(Bonga vs. Soler, 2 SCRA 755). (emphasis ours)
Our jurisprudence is therefore clear that (1) any disposition of estate property by an administrator or
prospective heir pending final adjudication requires court approval and (2) any unauthorized disposition
of estate property can be annulled by the probate court, there being no need for a separate action to annul
the unauthorized disposition.
The question now is: can the intestate or probate court execute its order nullifying the invalid sale?
We see no reason why it cannot. The intestate court has the power to execute its order with regard to
the nullity of an unauthorized sale of estate property, otherwise its power to annul the unauthorized or
fraudulent disposition of estate property would be meaningless. In other words, enforcement is a
necessary adjunct of the intestate or probate courts power to annul unauthorized or fraudulent transactions
to prevent the dissipation of estate property before final adjudication.
Moreover, in this case, the order of the intestate court nullifying the sale was affirmed by the
appellate courts (the Court of Appeals in CA-G.R. SP No. 46342 dated June 23, 1998 and subsequently
by the Supreme Court in G.R. No. 135177 dated October 9, 1998). The finality of the decision of the
Supreme Court was entered in the book of entry of judgments on February 23, 1999. Considering the
finality of the order of the intestate court nullifying the sale, as affirmed by the appellate courts, it was
correct for private respondent-Special Administratrix Enderes to thereafter move for a writ of execution
and for the intestate court to grant it.
Petitioners Jose Lee, Alma Aggabao and FLAG, however, contend that the probate court could not
issue a writ of execution with regard to its order nullifying the sale because said order was merely
provisional:
The only authority given by law is for respondent judge to determine provisionally whether said shares
are included or excluded in the inventory In ordering the execution of the orders, respondent judge acted
in excess of his jurisdiction and grossly violated settled law and jurisprudence, i.e., that the determination
by a probate or intestate court of whether a property is included or excluded in the inventory of the estate
being provisional in nature, cannot be the subject of execution.[24] (emphasis ours)
Petitioners argument is misplaced. There is no question, based on the facts of this case, that the
Philinterlife shares of stock were part of the estate of Dr. Juvencio Ortaez from the very start as in fact
these shares were included in the inventory of the properties of the estate submitted by Rafael Ortaez after
he and his brother, Jose Ortaez, were appointed special administrators by the intestate court.[25]

The controversy here actually started when, during the pendency of the settlement of the estate of Dr.
Ortaez, his wife Juliana Ortaez sold the 1,014 Philinterlife shares of stock in favor petitioner FLAG
without the approval of the intestate court. Her son Jose Ortaez later sold the remaining 1,011 Philinterlife
shares also in favor of FLAG without the approval of the intestate court.
We are not dealing here with the issue of inclusion or exclusion of properties in the inventory of the
estate because there is no question that, from the very start, the Philinterlife shares of stock were owned
by the decedent, Dr. Juvencio Ortaez. Rather, we are concerned here with the effect of the sale made
by the decedents heirs, Juliana Ortaez and Jose Ortaez, without the required approval of the
intestate court. This being so, the contention of petitioners that the determination of the intestate court
was merely provisional and should have been threshed out in a separate proceeding is incorrect.
The petitioners Jose Lee and Alma Aggabao next contend that the writ of execution should not be
executed against them because they were not notified, nor they were aware, of the proceedings nullifying
the sale of the shares of stock.
We are not persuaded. The title of the purchaser like herein petitioner FLAG can be struck down by
the intestate court after a clear showing of the nullity of the alienation. This is the logical consequence of
our ruling in Godoy and in several subsequent cases.[26] The sale of any property of the estate by an
administrator or prospective heir without order of the probate or intestate court is void and passes
no title to the purchaser. Thus, in Juan Lao et al. vs. Hon. Melencio Geneto, G.R. No. 56451, June 19,
1985, we ordered the probate court to cancel the transfer certificate of title issued to the vendees at the
instance of the administrator after finding that the sale of real property under probate proceedings was
made without the prior approval of the court. The dispositive portion of our decision read:
IN VIEW OF THE FOREGOING CONSIDERATIONS, the assailed Order dated February 18, 1981 of
the respondent Judge approving the questioned Amicable Settlement is declared NULL and VOID and
hereby SET ASIDE. Consequently, the sale in favor of Sotero Dioniosio III and by the latter to William
Go is likewise declared NULL and VOID. The Transfer Certificate of Title issued to the latter is hereby
ordered cancelled.
It goes without saying that the increase in Philinterlifes authorized capital stock, approved on the
vote of petitioners non-existent shareholdings and obviously calculated to make it difficult for Dr. Ortaezs
estate to reassume its controlling interest in Philinterlife, was likewise void ab initio.
Petitioners next argue that they were denied due process.
We do not think so.
The facts show that petitioners, for reasons known only to them, did not appeal the decision of the
intestate court nullifying the sale of shares of stock in their favor. Only the vendor, Jose Ortaez, appealed
the case. A careful review of the records shows that petitioners had actual knowledge of the estate
settlement proceedings and that they knew private respondent Enderes was questioning therein the sale to
them of the Philinterlife shares of stock.
It must be noted that private respondent-Special Administratrix Enderes filed before the intestate
court (RTC of Quezon City, Branch 85) a Motion to Declare Void Ab Initio Deeds of Sale of Philinterlife
Shares of Stock on March 22, 1996. But as early as 1994, petitioners already knew of the pending
settlement proceedings and that the shares they bought were under the administration by the intestate
court because private respondent Ma. Divina Ortaez-Enderes and her mother Ligaya Novicio had filed a
case against them at the Securities and Exchange Commission on November 7, 1994, docketed as SEC
No. 11-94-4909, for annulment of transfer of shares of stock, annulment of sale of corporate properties,
annulment of subscriptions on increased capital stocks, accounting, inspection of corporate books and
records and damages with prayer for a writ of preliminary injunction and/or temporary restraining

order.[27] In said case, Enderes and her mother questioned the sale of the aforesaid shares of stock to
petitioners. The SEC hearing officer in fact, in his resolution dated March 24, 1995, deferred to the
jurisdiction of the intestate court to rule on the validity of the sale of shares of stock sold to petitioners by
Jose Ortaez and Juliana Ortaez:
Petitioners also averred that. . . the Philinterlife shares of Dr. Juvencio Ortaez who died, in 1980, are part
of his estate which is presently the subject matter of an intestate proceeding of the RTC of Quezon City,
Branch 85. Although, private respondents [Jose Lee et al.] presented the documents of partition whereby
the foregoing share of stocks were allegedly partitioned and conveyed to Jose S. Ortaez who allegedly
assigned the same to the other private respondents, approval of the Court was not presented. Thus, the
assignments to the private respondents [Jose Lee et al.] of the subject shares of stocks are void.
xxxxxxxxx
With respect to the alleged extrajudicial partition of the shares of stock owned by the late Dr. Juvencio
Ortaez, we rule that the matter properly belongs to the jurisdiction of the regular court where the intestate
proceedings are currently pending.[28]
With this resolution of the SEC hearing officer dated as early as March 24, 1995 recognizing the
jurisdiction of the intestate court to determine the validity of the extrajudicial partition of the estate of Dr.
Ortaez and the subsequent sale by the heirs of the decedent of the Philinterlife shares of stock to
petitioners, how can petitioners claim that they were not aware of the intestate proceedings?
Futhermore, when the resolution of the SEC hearing officer reached the Supreme Court in 1996
(docketed as G.R. 128525), herein petitioners who were respondents therein filed their answer which
contained statements showing that they knew of the pending intestate proceedings:
[T]he subject matter of the complaint is not within the jurisdiction of the SEC but with the Regional Trial
Court; Ligaya Novicio and children represented themselves to be the common law wife and illegitimate
children of the late Ortaez; that on March 4, 1982, the surviving spouse Juliana Ortaez, on her behalf and
for her minor son Antonio, executed a Memorandum of Agreement with her other sons Rafael and Jose,
both surnamed Ortaez, dividing the estate of the deceased composed of his one-half (1/2) share in the
conjugal properties; that in the said Memorandum of Agreement, Jose S. Ortaez acquired as his share of
the estate the 1,329 shares of stock in Philinterlife; that on March 4, 1982, Juliana and Rafael assigned
their respective shares of stock in Philinterlife to Jose; that contrary to the contentions of petitioners,
private respondents Jose Lee, Carlos Lee, Benjamin Lee and Alma Aggabao became stockholders of
Philinterlife on March 23, 1983 when Jose S. Ortaez, the principal stockholder at that time, executed a
deed of sale of his shares of stock to private respondents; and that the right of petitioners to question the
Memorandum of Agreement and the acquisition of shares of stock of private respondent is barred by
prescription.[29]
Also, private respondent-Special Administratrix Enderes offered additional proof of actual
knowledge of the settlement proceedings by petitioners which petitioners never denied: (1) that
petitioners were represented by Atty. Ricardo Calimag previously hired by the mother of private
respondent Enderes to initiate cases against petitioners Jose Lee and Alma Aggaboa for the nullification
of the sale of the shares of stock but said counsel made a conflicting turn-around and appeared instead as
counsel of petitioners, and (2) that the deeds of sale executed between petitioners and the heirs of the
decedent (vendors Juliana Ortaez and Jose Ortaez) were acknowledged before Atty. Ramon Carpio who,
during the pendency of the settlement proceedings, filed a motion for the approval of the sale of
Philinterlife shares of stock to the Knights of Columbus Fraternal Association, Inc. (which motion was,

however, later abandoned).[30] All this sufficiently proves that petitioners, through their counsels, knew of
the pending settlement proceedings.
Finally, petitioners filed several criminal cases such as libel (Criminal Case No. 97-7179-81), grave
coercion (Criminal Case No. 84624) and robbery (Criminal Case No. Q-96-67919) against private
respondents mother Ligaya Novicio who was a director of Philinterlife,[31] all of which criminal cases
were related to the questionable sale to petitioners of the Philinterlife shares of stock.
Considering these circumstances, we cannot accept petitioners claim of denial of due process. The
essence of due process is the reasonable opportunity to be heard. Where the opportunity to be heard has
been accorded, there is no denial of due process.[32] In this case, petitioners knew of the pending instestate
proceedings for the settlement of Dr. Juvencio Ortaezs estate but for reasons they alone knew, they never
intervened. When the court declared the nullity of the sale, they did not bother to appeal. And when they
were notified of the motion for execution of the Orders of the intestate court, they ignored the same.
Clearly, petitioners alone should bear the blame.
Petitioners next contend that we are bound by our ruling in G.R. No. 128525 entitled Ma. Divina
Ortaez-Enderes vs. Court of Appeals, dated December 17, 1999, where we allegedly ruled that the
intestate court may not pass upon the title to a certain property for the purpose of determining whether the
same should or should not be included in the inventory but such determination is not conclusive and is
subject to final decision in a separate action regarding ownership which may be constituted by the parties.
We are not unaware of our decision in G.R. No. 128525. The issue therein was whether the Court of
Appeals erred in affirming the resolution of the SEC that Enderes et al. were not entitled to the issuance
of the writ of preliminary injunction. We ruled that the Court of Appeals was correct in affirming the
resolution of the SEC denying the issuance of the writ of preliminary injunction because injunction is not
designed to protect contingent rights. Said case did not rule on the issue of the validity of the sale of
shares of stock belonging to the decedents estate without court approval nor of the validity of the writ of
execution issued by the intestate court. G.R. No. 128525 clearly involved a different issue and it does not
therefore apply to the present case.
Petitioners and all parties claiming rights under them are hereby warned not to further delay the
execution of the Orders of the intestate court dated August 11 and August 29, 1997.
WHEREFORE, the petition is hereby DENIED. The decision of the Court of Appeals in CA-G.R.
S.P. No. 59736 dated July 26, 2000, dismissing petitioners petition for certiorari and affirming the July 6,
2000 order of the trial court which ordered the execution of its (trial courts) August 11 and 29, 1997
orders, is hereby AFFIRMED.
SO ORDERED.
Vitug, (Chairman), and Carpio-Morales, JJ., concur.
Sandoval-Gutierrez, J., no part.

G.R. No. L-43082

June 18, 1937

PABLO LORENZO, as trustee of the estate of Thomas Hanley, deceased, plaintiff-appellant,


vs.
JUAN POSADAS, JR., Collector of Internal Revenue, defendant-appellant.
Pablo Lorenzo and Delfin Joven for plaintiff-appellant.
Office of the Solicitor-General Hilado for defendant-appellant.

LAUREL, J.:
On October 4, 1932, the plaintiff Pablo Lorenzo, in his capacity as trustee of the estate of Thomas Hanley,
deceased, brought this action in the Court of First Instance of Zamboanga against the defendant, Juan
Posadas, Jr., then the Collector of Internal Revenue, for the refund of the amount of P2,052.74, paid by
the plaintiff as inheritance tax on the estate of the deceased, and for the collection of interst thereon at the
rate of 6 per cent per annum, computed from September 15, 1932, the date when the aforesaid tax was
[paid under protest. The defendant set up a counterclaim for P1,191.27 alleged to be interest due on the
tax in question and which was not included in the original assessment. From the decision of the Court of
First Instance of Zamboanga dismissing both the plaintiff's complaint and the defendant's counterclaim,
both parties appealed to this court.
It appears that on May 27, 1922, one Thomas Hanley died in Zamboanga, Zamboanga, leaving a will
(Exhibit 5) and considerable amount of real and personal properties. On june 14, 1922, proceedings for
the probate of his will and the settlement and distribution of his estate were begun in the Court of First
Instance of Zamboanga. The will was admitted to probate. Said will provides, among other things, as
follows:
4. I direct that any money left by me be given to my nephew Matthew Hanley.
5. I direct that all real estate owned by me at the time of my death be not sold or otherwise
disposed of for a period of ten (10) years after my death, and that the same be handled and
managed by the executors, and proceeds thereof to be given to my nephew, Matthew Hanley, at
Castlemore, Ballaghaderine, County of Rosecommon, Ireland, and that he be directed that the
same be used only for the education of my brother's children and their descendants.
6. I direct that ten (10) years after my death my property be given to the above mentioned
Matthew Hanley to be disposed of in the way he thinks most advantageous.
xxx

xxx

xxx

8. I state at this time I have one brother living, named Malachi Hanley, and that my nephew,
Matthew Hanley, is a son of my said brother, Malachi Hanley.
The Court of First Instance of Zamboanga considered it proper for the best interests of ther estate to
appoint a trustee to administer the real properties which, under the will, were to pass to Matthew Hanley
ten years after the two executors named in the will, was, on March 8, 1924, appointed trustee. Moore took
his oath of office and gave bond on March 10, 1924. He acted as trustee until February 29, 1932, when he
resigned and the plaintiff herein was appointed in his stead.
During the incumbency of the plaintiff as trustee, the defendant Collector of Internal Revenue, alleging
that the estate left by the deceased at the time of his death consisted of realty valued at P27,920 and
personalty valued at P1,465, and allowing a deduction of P480.81, assessed against the estate an
inheritance tax in the amount of P1,434.24 which, together with the penalties for deliquency in payment
consisting of a 1 per cent monthly interest from July 1, 1931 to the date of payment and a surcharge of 25
per cent on the tax, amounted to P2,052.74. On March 15, 1932, the defendant filed a motion in the
testamentary proceedings pending before the Court of First Instance of Zamboanga (Special proceedings
No. 302) praying that the trustee, plaintiff herein, be ordered to pay to the Government the said sum of
P2,052.74. The motion was granted. On September 15, 1932, the plaintiff paid said amount under protest,
notifying the defendant at the same time that unless the amount was promptly refunded suit would be

brought for its recovery. The defendant overruled the plaintiff's protest and refused to refund the said
amount hausted, plaintiff went to court with the result herein above indicated.
In his appeal, plaintiff contends that the lower court erred:
I. In holding that the real property of Thomas Hanley, deceased, passed to his instituted heir,
Matthew Hanley, from the moment of the death of the former, and that from the time, the latter
became the owner thereof.
II. In holding, in effect, that there was deliquency in the payment of inheritance tax due on the
estate of said deceased.
III. In holding that the inheritance tax in question be based upon the value of the estate upon the
death of the testator, and not, as it should have been held, upon the value thereof at the expiration
of the period of ten years after which, according to the testator's will, the property could be and
was to be delivered to the instituted heir.
IV. In not allowing as lawful deductions, in the determination of the net amount of the estate
subject to said tax, the amounts allowed by the court as compensation to the "trustees" and paid to
them from the decedent's estate.
V. In not rendering judgment in favor of the plaintiff and in denying his motion for new trial.
The defendant-appellant contradicts the theories of the plaintiff and assigns the following error besides:
The lower court erred in not ordering the plaintiff to pay to the defendant the sum of P1,191.27,
representing part of the interest at the rate of 1 per cent per month from April 10, 1924, to June
30, 1931, which the plaintiff had failed to pay on the inheritance tax assessed by the defendant
against the estate of Thomas Hanley.
The following are the principal questions to be decided by this court in this appeal: (a) When does the
inheritance tax accrue and when must it be satisfied? (b) Should the inheritance tax be computed on the
basis of the value of the estate at the time of the testator's death, or on its value ten years later? (c) In
determining the net value of the estate subject to tax, is it proper to deduct the compensation due to
trustees? (d) What law governs the case at bar? Should the provisions of Act No. 3606 favorable to the
tax-payer be given retroactive effect? (e) Has there been deliquency in the payment of the inheritance tax?
If so, should the additional interest claimed by the defendant in his appeal be paid by the estate? Other
points of incidental importance, raised by the parties in their briefs, will be touched upon in the course of
this opinion.
(a) The accrual of the inheritance tax is distinct from the obligation to pay the same. Section 1536 as
amended, of the Administrative Code, imposes the tax upon "every transmission by virtue of inheritance,
devise, bequest, giftmortis causa, or advance in anticipation of inheritance,devise, or bequest." The tax
therefore is upon transmission or the transfer or devolution of property of a decedent, made effective by
his death. (61 C. J., p. 1592.) It is in reality an excise or privilege tax imposed on the right to succeed to,
receive, or take property by or under a will or the intestacy law, or deed, grant, or gift to become operative
at or after death. Acording to article 657 of the Civil Code, "the rights to the succession of a person are
transmitted from the moment of his death." "In other words", said Arellano, C. J., ". . . the heirs succeed
immediately to all of the property of the deceased ancestor. The property belongs to the heirs at the
moment of the death of the ancestor as completely as if the ancestor had executed and delivered to them a

deed for the same before his death." (Bondad vs. Bondad, 34 Phil., 232. See also, Mijares vs. Nery, 3
Phil., 195; Suilong & Co., vs. Chio-Taysan, 12 Phil., 13; Lubrico vs. Arbado, 12 Phil., 391; Innocencio
vs. Gat-Pandan, 14 Phil., 491; Aliasas vs.Alcantara, 16 Phil., 489; Ilustre vs. Alaras Frondosa, 17 Phil.,
321; Malahacan vs. Ignacio, 19 Phil., 434; Bowa vs. Briones, 38 Phil., 27; Osario vs. Osario & Yuchausti
Steamship Co., 41 Phil., 531; Fule vs. Fule, 46 Phil., 317; Dais vs. Court of First Instance of Capiz, 51
Phil., 396; Baun vs. Heirs of Baun, 53 Phil., 654.) Plaintiff, however, asserts that while article 657 of the
Civil Code is applicable to testate as well as intestate succession, it operates only in so far as forced heirs
are concerned. But the language of article 657 of the Civil Code is broad and makes no distinction
between different classes of heirs. That article does not speak of forced heirs; it does not even use the
word "heir". It speaks of the rights of succession and the transmission thereof from the moment of death.
The provision of section 625 of the Code of Civil Procedure regarding the authentication and probate of a
will as a necessary condition to effect transmission of property does not affect the general rule laid down
in article 657 of the Civil Code. The authentication of a will implies its due execution but once probated
and allowed the transmission is effective as of the death of the testator in accordance with article 657 of
the Civil Code. Whatever may be the time when actual transmission of the inheritance takes place,
succession takes place in any event at the moment of the decedent's death. The time when the heirs legally
succeed to the inheritance may differ from the time when the heirs actually receive such inheritance.
"Poco importa", says Manresa commenting on article 657 of the Civil Code, "que desde el falleimiento
del causante, hasta que el heredero o legatario entre en posesion de los bienes de la herencia o del
legado, transcurra mucho o poco tiempo, pues la adquisicion ha de retrotraerse al momento de la
muerte, y asi lo ordena el articulo 989, que debe considerarse como complemento del presente." (5
Manresa, 305; see also, art. 440, par. 1, Civil Code.) Thomas Hanley having died on May 27, 1922, the
inheritance tax accrued as of the date.
From the fact, however, that Thomas Hanley died on May 27, 1922, it does not follow that the obligation
to pay the tax arose as of the date. The time for the payment on inheritance tax is clearly fixed by section
1544 of the Revised Administrative Code as amended by Act No. 3031, in relation to section 1543 of the
same Code. The two sections follow:
SEC. 1543. Exemption of certain acquisitions and transmissions. The following shall not be
taxed:
(a) The merger of the usufruct in the owner of the naked title.
(b) The transmission or delivery of the inheritance or legacy by the fiduciary heir or
legatee to the trustees.
(c) The transmission from the first heir, legatee, or donee in favor of another beneficiary,
in accordance with the desire of the predecessor.
In the last two cases, if the scale of taxation appropriate to the new beneficiary is greater than that
paid by the first, the former must pay the difference.
SEC. 1544. When tax to be paid. The tax fixed in this article shall be paid:
(a) In the second and third cases of the next preceding section, before entrance into
possession of the property.
(b) In other cases, within the six months subsequent to the death of the predecessor; but if
judicial testamentary or intestate proceedings shall be instituted prior to the expiration of

said period, the payment shall be made by the executor or administrator before delivering
to each beneficiary his share.
If the tax is not paid within the time hereinbefore prescribed, interest at the rate of twelve per
centum per annum shall be added as part of the tax; and to the tax and interest due and unpaid
within ten days after the date of notice and demand thereof by the collector, there shall be further
added a surcharge of twenty-five per centum.
A certified of all letters testamentary or of admisitration shall be furnished the Collector of
Internal Revenue by the Clerk of Court within thirty days after their issuance.
It should be observed in passing that the word "trustee", appearing in subsection (b) of section 1543,
should read "fideicommissary" or "cestui que trust". There was an obvious mistake in translation from the
Spanish to the English version.
The instant case does fall under subsection (a), but under subsection (b), of section 1544 above-quoted, as
there is here no fiduciary heirs, first heirs, legatee or donee. Under the subsection, the tax should have
been paid before the delivery of the properties in question to P. J. M. Moore as trustee on March 10, 1924.
(b) The plaintiff contends that the estate of Thomas Hanley, in so far as the real properties are concerned,
did not and could not legally pass to the instituted heir, Matthew Hanley, until after the expiration of ten
years from the death of the testator on May 27, 1922 and, that the inheritance tax should be based on the
value of the estate in 1932, or ten years after the testator's death. The plaintiff introduced evidence tending
to show that in 1932 the real properties in question had a reasonable value of only P5,787. This amount
added to the value of the personal property left by the deceased, which the plaintiff admits is P1,465,
would generate an inheritance tax which, excluding deductions, interest and surcharge, would amount
only to about P169.52.
If death is the generating source from which the power of the estate to impose inheritance taxes takes its
being and if, upon the death of the decedent, succession takes place and the right of the estate to tax vests
instantly, the tax should be measured by the vlaue of the estate as it stood at the time of the decedent's
death, regardless of any subsequent contingency value of any subsequent increase or decrease in value.
(61 C. J., pp. 1692, 1693; 26 R. C. L., p. 232; Blakemore and Bancroft, Inheritance Taxes, p. 137. See
also Knowlton vs. Moore, 178 U.S., 41; 20 Sup. Ct. Rep., 747; 44 Law. ed., 969.) "The right of the state
to an inheritance tax accrues at the moment of death, and hence is ordinarily measured as to any
beneficiary by the value at that time of such property as passes to him. Subsequent appreciation or
depriciation is immaterial." (Ross, Inheritance Taxation, p. 72.)
Our attention is directed to the statement of the rule in Cyclopedia of Law of and Procedure (vol. 37, pp.
1574, 1575) that, in the case of contingent remainders, taxation is postponed until the estate vests in
possession or the contingency is settled. This rule was formerly followed in New York and has been
adopted in Illinois, Minnesota, Massachusetts, Ohio, Pennsylvania and Wisconsin. This rule, horever, is
by no means entirely satisfactory either to the estate or to those interested in the property (26 R. C. L., p.
231.). Realizing, perhaps, the defects of its anterior system, we find upon examination of cases and
authorities that New York has varied and now requires the immediate appraisal of the postponed estate at
its clear market value and the payment forthwith of the tax on its out of the corpus of the estate
transferred. (In re Vanderbilt, 172 N. Y., 69; 69 N. E., 782; In re Huber, 86 N. Y. App. Div., 458; 83 N.
Y. Supp., 769; Estate of Tracy, 179 N. Y., 501; 72 N. Y., 519; Estate of Brez, 172 N. Y., 609; 64 N. E.,
958; Estate of Post, 85 App. Div., 611; 82 N. Y. Supp., 1079. Vide also, Saltoun vs. Lord Advocate, 1

Peter. Sc. App., 970; 3 Macq. H. L., 659; 23 Eng. Rul. Cas., 888.) California adheres to this new rule
(Stats. 1905, sec. 5, p. 343).
But whatever may be the rule in other jurisdictions, we hold that a transmission by inheritance is taxable
at the time of the predecessor's death, notwithstanding the postponement of the actual possession or
enjoyment of the estate by the beneficiary, and the tax measured by the value of the property transmitted
at that time regardless of its appreciation or depreciation.
(c) Certain items are required by law to be deducted from the appraised gross in arriving at the net value
of the estate on which the inheritance tax is to be computed (sec. 1539, Revised Administrative Code). In
the case at bar, the defendant and the trial court allowed a deduction of only P480.81. This sum represents
the expenses and disbursements of the executors until March 10, 1924, among which were their fees and
the proven debts of the deceased. The plaintiff contends that the compensation and fees of the trustees,
which aggregate P1,187.28 (Exhibits C, AA, EE, PP, HH, JJ, LL, NN, OO), should also be deducted
under section 1539 of the Revised Administrative Code which provides, in part, as follows: "In order to
determine the net sum which must bear the tax, when an inheritance is concerned, there shall be deducted,
in case of a resident, . . . the judicial expenses of the testamentary or intestate proceedings, . . . ."
A trustee, no doubt, is entitled to receive a fair compensation for his services (Barney vs. Saunders, 16
How., 535; 14 Law. ed., 1047). But from this it does not follow that the compensation due him may
lawfully be deducted in arriving at the net value of the estate subject to tax. There is no statute in the
Philippines which requires trustees' commissions to be deducted in determining the net value of the estate
subject to inheritance tax (61 C. J., p. 1705). Furthermore, though a testamentary trust has been created, it
does not appear that the testator intended that the duties of his executors and trustees should be separated.
(Ibid.; In re Vanneck's Estate, 161 N. Y. Supp., 893; 175 App. Div., 363; In re Collard's Estate, 161 N. Y.
Supp., 455.) On the contrary, in paragraph 5 of his will, the testator expressed the desire that his real
estate be handled and managed by his executors until the expiration of the period of ten years therein
provided. Judicial expenses are expenses of administration (61 C. J., p. 1705) but, in State vs. Hennepin
County Probate Court (112 N. W., 878; 101 Minn., 485), it was said: ". . . The compensation of a trustee,
earned, not in the administration of the estate, but in the management thereof for the benefit of the
legatees or devises, does not come properly within the class or reason for exempting administration
expenses. . . . Service rendered in that behalf have no reference to closing the estate for the purpose of a
distribution thereof to those entitled to it, and are not required or essential to the perfection of the rights of
the heirs or legatees. . . . Trusts . . . of the character of that here before the court, are created for the the
benefit of those to whom the property ultimately passes, are of voluntary creation, and intended for the
preservation of the estate. No sound reason is given to support the contention that such expenses should
be taken into consideration in fixing the value of the estate for the purpose of this tax."
(d) The defendant levied and assessed the inheritance tax due from the estate of Thomas Hanley under the
provisions of section 1544 of the Revised Administrative Code, as amended by section 3 of Act No. 3606.
But Act No. 3606 went into effect on January 1, 1930. It, therefore, was not the law in force when the
testator died on May 27, 1922. The law at the time was section 1544 above-mentioned, as amended by
Act No. 3031, which took effect on March 9, 1922.
It is well-settled that inheritance taxation is governed by the statute in force at the time of the death of the
decedent (26 R. C. L., p. 206; 4 Cooley on Taxation, 4th ed., p. 3461). The taxpayer can not foresee and
ought not to be required to guess the outcome of pending measures. Of course, a tax statute may be made
retroactive in its operation. Liability for taxes under retroactive legislation has been "one of the incidents
of social life." (Seattle vs. Kelleher, 195 U. S., 360; 49 Law. ed., 232 Sup. Ct. Rep., 44.) But legislative
intent that a tax statute should operate retroactively should be perfectly clear. (Scwab vs. Doyle, 42 Sup.

Ct. Rep., 491; Smietanka vs. First Trust & Savings Bank, 257 U. S., 602; Stockdale vs. Insurance Co., 20
Wall., 323; Lunch vs. Turrish, 247 U. S., 221.) "A statute should be considered as prospective in its
operation, whether it enacts, amends, or repeals an inheritance tax, unless the language of the statute
clearly demands or expresses that it shall have a retroactive effect, . . . ." (61 C. J., P. 1602.) Though the
last paragraph of section 5 of Regulations No. 65 of the Department of Finance makes section 3 of Act
No. 3606, amending section 1544 of the Revised Administrative Code, applicable to all estates the
inheritance taxes due from which have not been paid, Act No. 3606 itself contains no provisions
indicating legislative intent to give it retroactive effect. No such effect can begiven the statute by this
court.
The defendant Collector of Internal Revenue maintains, however, that certain provisions of Act No. 3606
are more favorable to the taxpayer than those of Act No. 3031, that said provisions are penal in nature
and, therefore, should operate retroactively in conformity with the provisions of article 22 of the Revised
Penal Code. This is the reason why he applied Act No. 3606 instead of Act No. 3031. Indeed, under Act
No. 3606, (1) the surcharge of 25 per cent is based on the tax only, instead of on both the tax and the
interest, as provided for in Act No. 3031, and (2) the taxpayer is allowed twenty days from notice and
demand by rthe Collector of Internal Revenue within which to pay the tax, instead of ten days only as
required by the old law.
Properly speaking, a statute is penal when it imposes punishment for an offense committed against the
state which, under the Constitution, the Executive has the power to pardon. In common use, however, this
sense has been enlarged to include within the term "penal statutes" all status which command or prohibit
certain acts, and establish penalties for their violation, and even those which, without expressly
prohibiting certain acts, impose a penalty upon their commission (59 C. J., p. 1110). Revenue laws,
generally, which impose taxes collected by the means ordinarily resorted to for the collection of taxes are
not classed as penal laws, although there are authorities to the contrary. (See Sutherland, Statutory
Construction, 361; Twine Co. vs. Worthington, 141 U. S., 468; 12 Sup. Ct., 55; Rice vs. U. S., 4 C. C. A.,
104; 53 Fed., 910; Com. vs. Standard Oil Co., 101 Pa. St., 150; State vs. Wheeler, 44 P., 430; 25 Nev.
143.) Article 22 of the Revised Penal Code is not applicable to the case at bar, and in the absence of clear
legislative intent, we cannot give Act No. 3606 a retroactive effect.
(e) The plaintiff correctly states that the liability to pay a tax may arise at a certain time and the tax may
be paid within another given time. As stated by this court, "the mere failure to pay one's tax does not
render one delinqent until and unless the entire period has eplased within which the taxpayer is authorized
by law to make such payment without being subjected to the payment of penalties for fasilure to pay his
taxes within the prescribed period." (U. S. vs. Labadan, 26 Phil., 239.)
The defendant maintains that it was the duty of the executor to pay the inheritance tax before the delivery
of the decedent's property to the trustee. Stated otherwise, the defendant contends that delivery to the
trustee was delivery to the cestui que trust, the beneficiery in this case, within the meaning of the first
paragraph of subsection (b) of section 1544 of the Revised Administrative Code. This contention is well
taken and is sustained. The appointment of P. J. M. Moore as trustee was made by the trial court in
conformity with the wishes of the testator as expressed in his will. It is true that the word "trust" is not
mentioned or used in the will but the intention to create one is clear. No particular or technical words are
required to create a testamentary trust (69 C. J., p. 711). The words "trust" and "trustee", though apt for
the purpose, are not necessary. In fact, the use of these two words is not conclusive on the question that a
trust is created (69 C. J., p. 714). "To create a trust by will the testator must indicate in the will his
intention so to do by using language sufficient to separate the legal from the equitable estate, and with
sufficient certainty designate the beneficiaries, their interest in the ttrust, the purpose or object of the trust,
and the property or subject matter thereof. Stated otherwise, to constitute a valid testamentary trust there

must be a concurrence of three circumstances: (1) Sufficient words to raise a trust; (2) a definite subject;
(3) a certain or ascertain object; statutes in some jurisdictions expressly or in effect so providing." (69 C.
J., pp. 705,706.) There is no doubt that the testator intended to create a trust. He ordered in his will that
certain of his properties be kept together undisposed during a fixed period, for a stated purpose. The
probate court certainly exercised sound judgment in appointment a trustee to carry into effect the
provisions of the will (see sec. 582, Code of Civil Procedure).
P. J. M. Moore became trustee on March 10, 1924. On that date trust estate vested in him (sec. 582 in
relation to sec. 590, Code of Civil Procedure). The mere fact that the estate of the deceased was placed in
trust did not remove it from the operation of our inheritance tax laws or exempt it from the payment of the
inheritance tax. The corresponding inheritance tax should have been paid on or before March 10, 1924, to
escape the penalties of the laws. This is so for the reason already stated that the delivery of the estate to
the trustee was in esse delivery of the same estate to the cestui que trust, the beneficiary in this case. A
trustee is but an instrument or agent for thecestui que trust (Shelton vs. King, 299 U. S., 90; 33 Sup. Ct.
Rep., 689; 57 Law. ed., 1086). When Moore accepted the trust and took possesson of the trust estate he
thereby admitted that the estate belonged not to him but to his cestui que trust (Tolentino vs. Vitug, 39
Phil.,126, cited in 65 C. J., p. 692, n. 63). He did not acquire any beneficial interest in the estate. He took
such legal estate only as the proper execution of the trust required (65 C. J., p. 528) and, his estate ceased
upon the fulfillment of the testator's wishes. The estate then vested absolutely in the beneficiary (65 C. J.,
p. 542).
The highest considerations of public policy also justify the conclusion we have reached. Were we to hold
that the payment of the tax could be postponed or delayed by the creation of a trust of the type at hand,
the result would be plainly disastrous. Testators may provide, as Thomas Hanley has provided, that their
estates be not delivered to their beneficiaries until after the lapse of a certain period of time. In the case at
bar, the period is ten years. In other cases, the trust may last for fifty years, or for a longer period which
does not offend the rule against petuities. The collection of the tax would then be left to the will of a
private individual. The mere suggestion of this result is a sufficient warning against the accpetance of the
essential to the very exeistence of government. (Dobbins vs. Erie Country, 16 Pet., 435; 10 Law. ed.,
1022; Kirkland vs. Hotchkiss, 100 U. S., 491; 25 Law. ed., 558; Lane County vs. Oregon, 7 Wall., 71; 19
Law. ed., 101; Union Refrigerator Transit Co. vs. Kentucky, 199 U. S., 194; 26 Sup. Ct. Rep., 36; 50
Law. ed., 150; Charles River Bridge vs. Warren Bridge, 11 Pet., 420; 9 Law. ed., 773.) The obligation to
pay taxes rests not upon the privileges enjoyed by, or the protection afforded to, a citizen by the
government but upon the necessity of money for the support of the state (Dobbins vs. Erie
Country, supra). For this reason, no one is allowed to object to or resist the payment of taxes solely
because no personal benefit to him can be pointed out. (Thomas vs. Gay, 169 U. S., 264; 18 Sup. Ct.
Rep., 340; 43 Law. ed., 740.) While courts will not enlarge, by construction, the government's power of
taxation (Bromley vs. McCaughn, 280 U. S., 124; 74 Law. ed., 226; 50 Sup. Ct. Rep., 46) they also will
not place upon tax laws so loose a construction as to permit evasions on merely fanciful and insubstantial
distictions. (U. S. vs. Watts, 1 Bond., 580; Fed. Cas. No. 16,653; U. S. vs. Wigglesirth, 2 Story, 369; Fed.
Cas. No. 16,690, followed in Froelich & Kuttner vs. Collector of Customs, 18 Phil., 461, 481; Castle
Bros., Wolf & Sons vs. McCoy, 21 Phil., 300; Muoz & Co. vs. Hord, 12 Phil., 624; Hongkong &
Shanghai Banking Corporation vs. Rafferty, 39 Phil., 145; Luzon Stevedoring Co. vs. Trinidad, 43 Phil.,
803.) When proper, a tax statute should be construed to avoid the possibilities of tax evasion. Construed
this way, the statute, without resulting in injustice to the taxpayer, becomes fair to the government.
That taxes must be collected promptly is a policy deeply intrenched in our tax system. Thus, no court is
allowed to grant injunction to restrain the collection of any internal revenue tax ( sec. 1578, Revised
Administrative Code; Sarasola vs. Trinidad, 40 Phil., 252). In the case of Lim Co Chui vs. Posadas (47
Phil., 461), this court had occassion to demonstrate trenchment adherence to this policy of the law. It held

that "the fact that on account of riots directed against the Chinese on October 18, 19, and 20, 1924, they
were prevented from praying their internal revenue taxes on time and by mutual agreement closed their
homes and stores and remained therein, does not authorize the Collector of Internal Revenue to extend the
time prescribed for the payment of the taxes or to accept them without the additional penalty of twenty
five per cent." (Syllabus, No. 3.)
". . . It is of the utmost importance," said the Supreme Court of the United States, ". . . that the modes
adopted to enforce the taxes levied should be interfered with as little as possible. Any delay in the
proceedings of the officers, upon whom the duty is developed of collecting the taxes, may derange the
operations of government, and thereby, cause serious detriment to the public." (Dows vs. Chicago, 11
Wall., 108; 20 Law. ed., 65, 66; Churchill and Tait vs. Rafferty, 32 Phil., 580.)
It results that the estate which plaintiff represents has been delinquent in the payment of inheritance tax
and, therefore, liable for the payment of interest and surcharge provided by law in such cases.
The delinquency in payment occurred on March 10, 1924, the date when Moore became trustee. The
interest due should be computed from that date and it is error on the part of the defendant to compute it
one month later. The provisions cases is mandatory (see and cf. Lim Co Chui vs. Posadas, supra), and
neither the Collector of Internal Revenuen or this court may remit or decrease such interest, no matter
how heavily it may burden the taxpayer.
To the tax and interest due and unpaid within ten days after the date of notice and demand thereof by the
Collector of Internal Revenue, a surcharge of twenty-five per centum should be added (sec. 1544, subsec.
(b), par. 2, Revised Administrative Code). Demand was made by the Deputy Collector of Internal
Revenue upon Moore in a communiction dated October 16, 1931 (Exhibit 29). The date fixed for the
payment of the tax and interest was November 30, 1931. November 30 being an official holiday, the tenth
day fell on December 1, 1931. As the tax and interest due were not paid on that date, the estate became
liable for the payment of the surcharge.
In view of the foregoing, it becomes unnecessary for us to discuss the fifth error assigned by the plaintiff
in his brief.
We shall now compute the tax, together with the interest and surcharge due from the estate of Thomas
Hanley inaccordance with the conclusions we have reached.
At the time of his death, the deceased left real properties valued at P27,920 and personal properties worth
P1,465, or a total of P29,385. Deducting from this amount the sum of P480.81, representing allowable
deductions under secftion 1539 of the Revised Administrative Code, we have P28,904.19 as the net value
of the estate subject to inheritance tax.
The primary tax, according to section 1536, subsection (c), of the Revised Administrative Code, should
be imposed at the rate of one per centum upon the first ten thousand pesos and two per centum upon the
amount by which the share exceed thirty thousand pesos, plus an additional two hundred per centum. One
per centum of ten thousand pesos is P100. Two per centum of P18,904.19 is P378.08. Adding to these
two sums an additional two hundred per centum, or P965.16, we have as primary tax, correctly computed
by the defendant, the sum of P1,434.24.
To the primary tax thus computed should be added the sums collectible under section 1544 of the Revised
Administrative Code. First should be added P1,465.31 which stands for interest at the rate of twelve per
centum per annum from March 10, 1924, the date of delinquency, to September 15, 1932, the date of

payment under protest, a period covering 8 years, 6 months and 5 days. To the tax and interest thus
computed should be added the sum of P724.88, representing a surhcarge of 25 per cent on both the tax
and interest, and also P10, the compromise sum fixed by the defendant (Exh. 29), giving a grand total of
P3,634.43.
As the plaintiff has already paid the sum of P2,052.74, only the sums of P1,581.69 is legally due from the
estate. This last sum is P390.42 more than the amount demanded by the defendant in his counterclaim.
But, as we cannot give the defendant more than what he claims, we must hold that the plaintiff is liable
only in the sum of P1,191.27 the amount stated in the counterclaim.
The judgment of the lower court is accordingly modified, with costs against the plaintiff in both instances.
So ordered.
Avancea, C.J., Abad Santos, Imperial, Diaz and Concepcion, JJ., concur.
Villa-Real, J., concurs.

G.R. No. 82027 March 29, 1990


ROMARICO G. VITUG, petitioner,
vs.
THE HONORABLE COURT OF APPEALS and ROWENA FAUSTINO-CORONA, respondents.
Rufino B. Javier Law Office for petitioner.
Quisumbing, Torres & Evangelista for private respondent.

SARMIENTO, J.:
This case is a chapter in an earlier suit decided by this Court 1 involving the probate of the two wills of the
late Dolores Luchangco Vitug, who died in New York, U. S.A., on November 10, 1980, naming private
respondent Rowena Faustino-Corona executrix. In our said decision, we upheld the appointment of
Nenita Alonte as co-special administrator of Mrs. Vitug's estate with her (Mrs. Vitug's) widower,
petitioner Romarico G. Vitug, pending probate.
On January 13, 1985, Romarico G. Vitug filed a motion asking for authority from the probate court to sell
certain shares of stock and real properties belonging to the estate to cover allegedly his advances to the
estate in the sum of P667,731.66, plus interests, which he claimed were personal funds. As found by the
Court of Appeals, 2the alleged advances consisted of P58,147.40 spent for the payment of estate tax,
P518,834.27 as deficiency estate tax, and P90,749.99 as "increment thereto." 3 According to Mr. Vitug, he
withdrew the sums of P518,834.27 and P90,749.99 from savings account No. 35342-038 of the Bank of
America, Makati, Metro Manila.
On April 12, 1985, Rowena Corona opposed the motion to sell on the ground that the same funds
withdrawn from savings account No. 35342-038 were conjugal partnership properties and part of the

estate, and hence, there was allegedly no ground for reimbursement. She also sought his ouster for failure
to include the sums in question for inventory and for "concealment of funds belonging to the estate." 4
Vitug insists that the said funds are his exclusive property having acquired the same through a
survivorship agreement executed with his late wife and the bank on June 19, 1970. The agreement
provides:
We hereby agree with each other and with the BANK OF AMERICAN NATIONAL
TRUST AND SAVINGS ASSOCIATION (hereinafter referred to as the BANK), that all
money now or hereafter deposited by us or any or either of us with the BANK in our joint
savings current account shall be the property of all or both of us and shall be payable to
and collectible or withdrawable by either or any of us during our lifetime, and after the
death of either or any of us shall belong to and be the sole property of the survivor or
survivors, and shall be payable to and collectible or withdrawable by such survivor or
survivors.
We further agree with each other and the BANK that the receipt or check of either, any or
all of us during our lifetime, or the receipt or check of the survivor or survivors, for any
payment or withdrawal made for our above-mentioned account shall be valid and
sufficient release and discharge of the BANK for such payment or withdrawal. 5
The trial courts 6 upheld the validity of this agreement and granted "the motion to sell some of the estate
of Dolores L. Vitug, the proceeds of which shall be used to pay the personal funds of Romarico Vitug in
the total sum of P667,731.66 ... ."7
On the other hand, the Court of Appeals, in the petition for certiorari filed by the herein private
respondent, held that the above-quoted survivorship agreement constitutes a conveyance mortis
causa which "did not comply with the formalities of a valid will as prescribed by Article 805 of the Civil
Code," 8 and secondly, assuming that it is a mere donation inter vivos, it is a prohibited donation under the
provisions of Article 133 of the Civil Code. 9
The dispositive portion of the decision of the Court of Appeals states:
WHEREFORE, the order of respondent Judge dated November 26, 1985 (Annex II,
petition) is hereby set aside insofar as it granted private respondent's motion to sell
certain properties of the estate of Dolores L. Vitug for reimbursement of his alleged
advances to the estate, but the same order is sustained in all other respects. In addition,
respondent Judge is directed to include provisionally the deposits in Savings Account No.
35342-038 with the Bank of America, Makati, in the inventory of actual properties
possessed by the spouses at the time of the decedent's death. With costs against private
respondent. 10
In his petition, Vitug, the surviving spouse, assails the appellate court's ruling on the strength of our
decisions inRivera v. People's Bank and Trust Co. 11 and Macam v. Gatmaitan 12 in which we sustained
the validity of "survivorship agreements" and considering them as aleatory contracts. 13
The petition is meritorious.
The conveyance in question is not, first of all, one of mortis causa, which should be embodied in a will. A
will has been defined as "a personal, solemn, revocable and free act by which a capacitated person

disposes of his property and rights and declares or complies with duties to take effect after his
death." 14 In other words, the bequest or device must pertain to the testator. 15 In this case, the monies
subject of savings account No. 35342-038 were in the nature of conjugal funds In the case relied
on, Rivera v. People's Bank and Trust Co., 16 we rejected claims that a survivorship agreement purports to
deliver one party's separate properties in favor of the other, but simply, their joint holdings:
xxx xxx xxx
... Such conclusion is evidently predicated on the assumption that Stephenson was the
exclusive owner of the funds-deposited in the bank, which assumption was in turn based
on the facts (1) that the account was originally opened in the name of Stephenson alone
and (2) that Ana Rivera "served only as housemaid of the deceased." But it not
infrequently happens that a person deposits money in the bank in the name of another;
and in the instant case it also appears that Ana Rivera served her master for about
nineteen years without actually receiving her salary from him. The fact that subsequently
Stephenson transferred the account to the name of himself and/or Ana Rivera and
executed with the latter the survivorship agreement in question although there was no
relation of kinship between them but only that of master and servant, nullifies the
assumption that Stephenson was the exclusive owner of the bank account. In the absence,
then, of clear proof to the contrary, we must give full faith and credit to the certificate of
deposit which recites in effect that the funds in question belonged to Edgar Stephenson
and Ana Rivera; that they were joint (and several) owners thereof; and that either of them
could withdraw any part or the whole of said account during the lifetime of both, and the
balance, if any, upon the death of either, belonged to the survivor. 17
xxx xxx xxx
In Macam v. Gatmaitan, 18 it was held:
xxx xxx xxx
This Court is of the opinion that Exhibit C is an aleatory contract whereby, according to
article 1790 of the Civil Code, one of the parties or both reciprocally bind themselves to
give or do something as an equivalent for that which the other party is to give or do in
case of the occurrence of an event which is uncertain or will happen at an indeterminate
time. As already stated, Leonarda was the owner of the house and Juana of the Buick
automobile and most of the furniture. By virtue of Exhibit C, Juana would become the
owner of the house in case Leonarda died first, and Leonarda would become the owner of
the automobile and the furniture if Juana were to die first. In this manner Leonarda and
Juana reciprocally assigned their respective property to one another conditioned upon
who might die first, the time of death determining the event upon which the acquisition of
such right by the one or the other depended. This contract, as any other contract, is
binding upon the parties thereto. Inasmuch as Leonarda had died before Juana, the latter
thereupon acquired the ownership of the house, in the same manner as Leonarda would
have acquired the ownership of the automobile and of the furniture if Juana had died
first. 19
xxx xxx xxx

There is no showing that the funds exclusively belonged to one party, and hence it must be presumed to
be conjugal, having been acquired during the existence of the marita. relations. 20
Neither is the survivorship agreement a donation inter vivos, for obvious reasons, because it was to take
effect after the death of one party. Secondly, it is not a donation between the spouses because it involved
no conveyance of a spouse's own properties to the other.
It is also our opinion that the agreement involves no modification petition of the conjugal partnership, as
held by the Court of Appeals, 21 by "mere stipulation" 22 and that it is no "cloak" 23 to circumvent the law
on conjugal property relations. Certainly, the spouses are not prohibited by law to invest conjugal
property, say, by way of a joint and several bank account, more commonly denominated in banking
parlance as an "and/or" account. In the case at bar, when the spouses Vitug opened savings account No.
35342-038, they merely put what rightfully belonged to them in a money-making venture. They did not
dispose of it in favor of the other, which would have arguably been sanctionable as a prohibited donation.
And since the funds were conjugal, it can not be said that one spouse could have pressured the other in
placing his or her deposits in the money pool.
The validity of the contract seems debatable by reason of its "survivor-take-all" feature, but in reality, that
contract imposed a mere obligation with a term, the term being death. Such agreements are permitted by
the Civil Code.24
Under Article 2010 of the Code:
ART. 2010. By an aleatory contract, one of the parties or both reciprocally bind
themselves to give or to do something in consideration of what the other shall give or do
upon the happening of an event which is uncertain, or which is to occur at an
indeterminate time.
Under the aforequoted provision, the fulfillment of an aleatory contract depends on either the happening
of an event which is (1) "uncertain," (2) "which is to occur at an indeterminate time." A survivorship
agreement, the sale of a sweepstake ticket, a transaction stipulating on the value of currency, and
insurance have been held to fall under the first category, while a contract for life annuity or pension under
Article 2021, et sequentia, has been categorized under the second. 25 In either case, the element of risk is
present. In the case at bar, the risk was the death of one party and survivorship of the other.
However, as we have warned:
xxx xxx xxx
But although the survivorship agreement is per se not contrary to law its operation or
effect may be violative of the law. For instance, if it be shown in a given case that such
agreement is a mere cloak to hide an inofficious donation, to transfer property in fraud of
creditors, or to defeat the legitime of a forced heir, it may be assailed and annulled upon
such grounds. No such vice has been imputed and established against the agreement
involved in this case. 26
xxx xxx xxx

There is no demonstration here that the survivorship agreement had been executed for such unlawful
purposes, or, as held by the respondent court, in order to frustrate our laws on wills, donations, and
conjugal partnership.
The conclusion is accordingly unavoidable that Mrs. Vitug having predeceased her husband, the latter has
acquired upon her death a vested right over the amounts under savings account No. 35342-038 of the
Bank of America. Insofar as the respondent court ordered their inclusion in the inventory of assets left by
Mrs. Vitug, we hold that the court was in error. Being the separate property of petitioner, it forms no more
part of the estate of the deceased.
WHEREFORE, the decision of the respondent appellate court, dated June 29, 1987, and its resolution,
dated February 9, 1988, are SET ASIDE.
No costs.
SO ORDERED.
Melencio-Herrera (Chairperson), Paras, Padilla and Regalado JJ., concur.

G.R. No. 122880

April 12, 2006

FELIX AZUELA, Petitioner,


vs.
COURT OF APPEALS, GERALDA AIDA CASTILLO substituted by ERNESTO G.
CASTILLO, Respondents.
DECISION
TINGA, J.:
The core of this petition is a highly defective notarial will, purportedly executed by Eugenia E. Igsolo
(decedent), who died on 16 December 1982 at the age of 80. In refusing to give legal recognition to the
due execution of this document, the Court is provided the opportunity to assert a few important doctrinal
rules in the execution of notarial wills, all self-evident in view of Articles 805 and 806 of the Civil Code.
A will whose attestation clause does not contain the number of pages on which the will is written is
fatally defective. A will whose attestation clause is not signed by the instrumental witnesses is fatally
defective. And perhaps most importantly, a will which does not contain an acknowledgment, but a
merejurat, is fatally defective. Any one of these defects is sufficient to deny probate. A notarial will
with all three defects is just aching for judicial rejection.
There is a distinct and consequential reason the Civil Code provides a comprehensive catalog of
imperatives for the proper execution of a notarial will. Full and faithful compliance with all the detailed
requisites under Article 805 of the Code leave little room for doubt as to the validity in the due execution
of the notarial will. Article 806 likewise imposes another safeguard to the validity of notarial wills that

they be acknowledged before a notary public by the testator and the witnesses. A notarial will executed
with indifference to these two codal provisions opens itself to nagging questions as to its legitimacy.
The case stems from a petition for probate filed on 10 April 1984 with the Regional Trial Court (RTC) of
Manila. The petition filed by petitioner Felix Azuela sought to admit to probate the notarial will of
Eugenia E. Igsolo, which was notarized on 10 June 1981. Petitioner is the son of the cousin of the
decedent.
The will, consisting of two (2) pages and written in the vernacular Pilipino, read in full:
HULING HABILIN NI EUGENIA E. IGSOLO
SA NGALAN NG MAYKAPAL, AMEN:
AKO, si EUGENIA E. IGSOLO, nakatira sa 500 San Diego St., Sampaloc, Manila, pitongput siyam (79)
na gulang, nasa hustong pagi-isip, pag-unawa at memoria ay nag-hahayag na ito na ang aking huling
habilin at testamento, at binabali wala ko lahat ang naunang ginawang habilin o testamento:
Una-Hinihiling ko na ako ay mailibing sa Sementerio del Norte, La Loma sang-ayong sa kaugalian at
patakaran ng simbahang katoliko at ang taga-pag-ingat (Executor) ng habiling ito ay magtatayo ng
bantayog upang silbing ala-ala sa akin ng aking pamilya at kaibigan;
Pangalawa-Aking ipinagkakaloob at isinasalin ang lahat ng karapatan sa aking pamangkin na si Felix
Azuela, na siyang nag-alaga sa akin sa mahabang panahon, yaong mga bahay na nakatirik sa lote numero
28, Block 24 at nakapangalan sa Pechaten Korporasyon, ganoon din ibinibigay ko ang lahat ng karapatan
sa bahay na nakatirik sa inoopahan kong lote, numero 43, Block 24 na pag-aari ng Pechaten Corporation.
Ipinagkakaloob kong buong buo ang lahat ng karapatan sa bahay at lupa na nasa 500 San Diego St., Lot
42, Block 24, Sampaloc, Manila kay Felix Azuela at ang pagkakaloob kong ito ay walang pasubalit at
kondiciones;
Pangatlo- Na ninunumbrahan ko si VART PAGUE na siyang nagpapatupad ng huling habiling ito at
kagustuhan ko rin na hindi na kailanman siyang mag-lagak ng piyansiya.
Aking nilagdaan ang Huling Habilin na ito dito sa Maynila ika 10 ng Hunyo, 1981.
(Sgd.)
EUGENIA E. IGSOLO
(Tagapagmana)
PATUNAY NG MGA SAKSI
Ang kasulatang ito, na binubuo ng ____ dahon pati ang huling dahong ito, na ipinahayag sa amin ni
Eugenia E. Igsolo, tagapagmana na siya niyang Huling Habilin, ngayon ika-10 ng Hunyo 1981, ay
nilagdaan ng nasabing tagapagmana sa ilalim ng kasulatang nabanggit at sa kaliwang panig ng lahat at
bawat dahon, sa harap ng lahat at bawat sa amin, at kami namang mga saksi ay lumagda sa harap ng
nasabing tagapagmana at sa harap ng lahat at bawat isa sa amin, sa ilalim ng nasabing kasulatan at sa
kaliwang panig ng lahat at bawat dahon ng kasulatan ito.

EUGENIA E. IGSOLO
address: 500 San Diego St.
Sampaloc, Manila Res. Cert. No. A-7717-37
Issued at Manila on March 10, 1981.
QUIRINO AGRAVA
address: 1228-Int. 3, Kahilum
Pandacan, Manila Res. Cert. No. A-458365
Issued at Manila on Jan. 21, 1981
LAMBERTO C. LEAO
address: Avenue 2, Blcok 7,
Lot 61, San Gabriel, G.MA., Cavite Res.
Cert. No. A-768277 issued at Carmona, Cavite on Feb. 7, 1981
JUANITO ESTRERA
address: City Court Compound,
City of Manila Res. Cert. No. A574829
Issued at Manila on March 2, 1981.
Nilagdaan ko at ninotario ko ngayong 10 ng Hunyo 10, 1981 dito sa Lungsod ng Maynila.
(Sgd.)
PETRONIO Y. BAUTISTA
Doc. No. 1232 ; NOTARIO PUBLIKO
Page No. 86 ; Until Dec. 31, 1981
Book No. 43 ; PTR-152041-1/2/81-Manila
Series of 1981 TAN # 1437-977-81
The three named witnesses to the will affixed their signatures on the left-hand margin of both pages of the
will, but not at the bottom of the attestation clause.
The probate petition adverted to only two (2) heirs, legatees and devisees of the decedent, namely:
petitioner himself, and one Irene Lynn Igsolo, who was alleged to have resided abroad. Petitioner prayed
that the will be allowed, and that letters testamentary be issued to the designated executor, Vart Prague.
The petition was opposed by Geralda Aida Castillo (Geralda Castillo), who represented herself as the
attorney-in-fact of "the 12 legitimate heirs" of the decedent.2 Geralda Castillo claimed that the will is a
forgery, and that the true purpose of its emergence was so it could be utilized as a defense in several court
cases filed by oppositor against petitioner, particularly for forcible entry and usurpation of real property,
all centering on petitioners right to occupy the properties of the decedent.3 It also asserted that contrary to
the representations of petitioner, the decedent was actually survived by 12 legitimate heirs, namely her
grandchildren, who were then residing abroad. Per records, it was subsequently alleged that decedent was
the widow of Bonifacio Igsolo, who died in 1965,4 and the mother of a legitimate child, Asuncion E.
Igsolo, who predeceased her mother by three (3) months.5
Oppositor Geralda Castillo also argued that the will was not executed and attested to in accordance with
law. She pointed out that decedents signature did not appear on the second page of the will, and the will
was not properly acknowledged. These twin arguments are among the central matters to this petition.

After due trial, the RTC admitted the will to probate, in an Order dated 10 August 1992.6 The RTC
favorably took into account the testimony of the three (3) witnesses to the will, Quirino Agrava, Lamberto
Leano, and Juanito Estrada. The RTC also called to fore "the modern tendency in respect to the
formalities in the execution of a will x x x with the end in view of giving the testator more freedom in
expressing his last wishes;"7 and from this perspective, rebutted oppositors arguments that the will was
not properly executed and attested to in accordance with law.
After a careful examination of the will and consideration of the testimonies of the subscribing and
attesting witnesses, and having in mind the modern tendency in respect to the formalities in the execution
of a will, i.e., the liberalization of the interpretation of the law on the formal requirements of a will with
the end in view of giving the testator more freedom in expressing his last wishes, this Court is persuaded
to rule that the will in question is authentic and had been executed by the testatrix in accordance with law.
On the issue of lack of acknowledgement, this Court has noted that at the end of the will after the
signature of the testatrix, the following statement is made under the sub-title, "Patunay Ng Mga Saksi":
"Ang kasulatang ito, na binubuo ng _____ dahon pati ang huling dahong ito, na ipinahayag sa amin ni
Eugenia N. Igsolo, tagapagmana na siya niyang Huling Habilin, ngayong ika-10 ng Hunyo 1981, ay
nilagdaan ng nasabing tagapagmana sa ilalim ng kasulatang nabanggit at sa kaliwang panig ng lahat at
bawat dahon, sa harap ng lahat at bawat sa amin, at kami namang mga saksi ay lumagda sa harap ng
nasabing tagapagmana at sa harap ng lahat at bawat isa sa amin, sa ilalim ng nasabing kasulatan at sa
kaliwang panig ng lahat at bawat dahon ng kasulatan ito."
The aforequoted declaration comprises the attestation clause and the acknowledgement and is considered
by this Court as a substantial compliance with the requirements of the law.
On the oppositors contention that the attestation clause was not signed by the subscribing witnesses at
the bottom thereof, this Court is of the view that the signing by the subscribing witnesses on the left
margin of the second page of the will containing the attestation clause and acknowledgment, instead of at
the bottom thereof, substantially satisfies the purpose of identification and attestation of the will.
With regard to the oppositors argument that the will was not numbered correlatively in letters placed on
upper part of each page and that the attestation did not state the number of pages thereof, it is worthy to
note that the will is composed of only two pages. The first page contains the entire text of the
testamentary dispositions, and the second page contains the last portion of the attestation clause and
acknowledgement. Such being so, the defects are not of a serious nature as to invalidate the will. For the
same reason, the failure of the testatrix to affix her signature on the left margin of the second page, which
contains only the last portion of the attestation clause and acknowledgment is not a fatal defect.
As regards the oppositors assertion that the signature of the testatrix on the will is a forgery, the
testimonies of the three subscribing witnesses to the will are convincing enough to establish the
genuineness of the signature of the testatrix and the due execution of the will.8
The Order was appealed to the Court of Appeals by Ernesto Castillo, who had substituted his since
deceased mother-in-law, Geralda Castillo. In a Decision dated 17 August 1995, the Court of Appeals
reversed the trial court and ordered the dismissal of the petition for probate.9 The Court of Appeals noted
that the attestation clause failed to state the number of pages used in the will, thus rendering the will void
and undeserving of probate.10
Hence, the present petition.

Petitioner argues that the requirement under Article 805 of the Civil Code that "the number of pages used
in a notarial will be stated in the attestation clause" is merely directory, rather than mandatory, and thus
susceptible to what he termed as "the substantial compliance rule."11
The solution to this case calls for the application of Articles 805 and 806 of the Civil Code, which we
replicate in full.
Art. 805. Every will, other than a holographic will, must be subscribed at the end thereof by the testator
himself or by the testator's name written by some other person in his presence, and by his express
direction, and attested and subscribed by three or more credible witnesses in the presence of the testator
and of one another.
The testator or the person requested by him to write his name and the instrumental witnesses of the will,
shall also sign, as aforesaid, each and every page thereof, except the last, on the left margin, and all the
pages shall be numbered correlatively in letters placed on the upper part of each page.
The attestation shall state the number of pages used upon which the will is written, and the fact that the
testator signed the will and every page thereof, or caused some other person to write his name, under his
express direction, in the presence of the instrumental witnesses, and that the latter witnessed and signed
the will and all the pages thereof in the presence of the testator and of one another.
If the attestation clause is in a language not known to the witnesses, it shall be interpreted to them.
Art. 806. Every will must be acknowledged before a notary public by the testator and the witnesses. The
notary public shall not be required to retain a copy of the will, or file another with the office of the Clerk
of Court.
The appellate court, in its Decision, considered only one defect, the failure of the attestation clause to
state the number of pages of the will. But an examination of the will itself reveals several more
deficiencies.
As admitted by petitioner himself, the attestation clause fails to state the number of pages of the
will.12 There was an incomplete attempt to comply with this requisite, a space having been allotted for the
insertion of the number of pages in the attestation clause. Yet the blank was never filled in; hence, the
requisite was left uncomplied with.
The Court of Appeals pounced on this defect in reversing the trial court, citing in the process Uy Coque v.
Navas L. Sioca13 and In re: Will of Andrada.14 In Uy Coque, the Court noted that among the defects of the
will in question was the failure of the attestation clause to state the number of pages contained in the
will.15 In ruling that the will could not be admitted to probate, the Court made the following consideration
which remains highly relevant to this day: "The purpose of requiring the number of sheets to be stated in
the attestation clause is obvious; the document might easily be so prepared that the removal of a sheet
would completely change the testamentary dispositions of the will and in the absence of a statement
of the total number of sheets such removal might be effected by taking out the sheet and changing
the numbers at the top of the following sheets or pages. If, on the other hand, the total number of
sheets is stated in the attestation clause the falsification of the document will involve the inserting of new
pages and the forging of the signatures of the testator and witnesses in the margin, a matter attended with
much greater difficulty."16

The case of In re Will of Andrada concerned a will the attestation clause of which failed to state the
number of sheets or pages used. This consideration alone was sufficient for the Court to declare
"unanim[ity] upon the point that the defect pointed out in the attesting clause is fatal."17 It was further
observed that "it cannot be denied that the x x x requirement affords additional security against the danger
that the will may be tampered with; and as the Legislature has seen fit to prescribe this requirement, it
must be considered material."18
Against these cited cases, petitioner cites Singson v. Florentino19 and Taboada v. Hon. Rosal,20 wherein
the Court allowed probate to the wills concerned therein despite the fact that the attestation clause did not
state the number of pages of the will. Yet the appellate court itself considered the import of these two
cases, and made the following distinction which petitioner is unable to rebut, and which we adopt with
approval:
Even a cursory examination of the Will (Exhibit "D"), will readily show that the attestation does not state
the number of pages used upon which the will is written. Hence, the Will is void and undeserving of
probate.
We are not impervious of the Decisions of the Supreme Court in "Manuel Singson versus Emilia
Florentino, et al., 92 Phil. 161 and Apolonio [Taboada] versus Hon. Avelino Rosal, et al., 118 SCRA
195," to the effect that a will may still be valid even if the attestation does not contain the number of
pages used upon which the Will is written. However, the Decisions of the Supreme Court are not
applicable in the aforementioned appeal at bench. This is so because, in the case of "Manuel Singson
versus Emilia Florentino, et al., supra," although the attestation in the subject Will did not state the
number of pages used in the will, however, the same was found in the last part of the body of the Will:
"x x x
The law referred to is article 618 of the Code of Civil Procedure, as amended by Act No. 2645, which
requires that the attestation clause shall state the number of pages or sheets upon which the will is written,
which requirement has been held to be mandatory as an effective safeguard against the possibility of
interpolation or omission of some of the pages of the will to the prejudice of the heirs to whom the
property is intended to be bequeathed (In re Will of Andrada, 42 Phil. 180; Uy Coque vs. Navas L. Sioca,
43 Phil., 405; Gumban vs. Gorcho, 50 Phil. 30; Quinto vs. Morata, 54 Phil. 481; Echevarria vs.
Sarmiento, 66 Phil. 611). The ratio decidendi of these cases seems to be that the attestation clause must
contain a statement of the number of sheets or pages composing the will and that if this is missing or is
omitted, it will have the effect of invalidating the will if the deficiency cannot be supplied, not by
evidence aliunde, but by a consideration or examination of the will itself. But here the situation is
different. While the attestation clause does not state the number of sheets or pages upon which the will is
written, however, the last part of the body of the will contains a statement that it is composed of eight
pages, which circumstance in our opinion takes this case out of the rigid rule of construction and places it
within the realm of similar cases where a broad and more liberal view has been adopted to prevent the
will of the testator from being defeated by purely technical considerations." (page 165-165, supra)
(Underscoring supplied)
In "Apolonio Tabaoda versus Hon. Avelino Rosal, et al." supra, the notarial acknowledgement in the Will
states the number of pages used in the:
"x x x

We have examined the will in question and noticed that the attestation clause failed to state the number of
pages used in writing the will. This would have been a fatal defect were it not for the fact that, in this
case, it is discernible from the entire will that it is really and actually composed of only two pages duly
signed by the testatrix and her instrumental witnesses. As earlier stated, the first page which contains the
entirety of the testamentary dispositions is signed by the testatrix at the end or at the bottom while the
instrumental witnesses signed at the left margin. The other page which is marked as "Pagina dos"
comprises the attestation clause and the acknowledgment. The acknowledgment itself states that "this
Last Will and Testament consists of two pages including this page" (pages 200-201, supra) (Underscoring
supplied).
However, in the appeal at bench, the number of pages used in the will is not stated in any part of the Will.
The will does not even contain any notarial acknowledgment wherein the number of pages of the will
should be stated.21
Both Uy Coque and Andrada were decided prior to the enactment of the Civil Code in 1950, at a time
when the statutory provision governing the formal requirement of wills was Section
618 of the Code of Civil Procedure.22 Reliance on these cases remains apropos, considering that the
requirement that the attestation state the number of pages of the will is extant from Section
618.23 However, the enactment of the Civil Code in 1950 did put in force a rule of interpretation of the
requirements of wills, at least insofar as the attestation clause is concerned, that may vary from the
philosophy that governed these two cases. Article 809 of the Civil Code states: "In the absence of bad
faith, forgery, or fraud, or undue and improper pressure and influence, defects and imperfections in the
form of attestation or in the language used therein shall not render the will invalid if it is proved that the
will was in fact executed and attested in substantial compliance with all the requirements of article 805."
In the same vein, petitioner cites the report of the Civil Code Commission, which stated that "the
underlying and fundamental objective permeating the provisions on the [law] on [wills] in this project
consists in the [liberalization] of the manner of their execution with the end in view of giving the testator
more [freedom] in [expressing] his last wishes. This objective is in accord with the [modern tendency] in
respect to the formalities in the execution of wills."24 However, petitioner conveniently omits the
qualification offered by the Code Commission in the very same paragraph he cites from their report, that
such liberalization be "but with sufficient safeguards and restrictions to prevent the commission of fraud
and the exercise of undue and improper pressure and influence upon the testator."25
Caneda v. Court of Appeals26 features an extensive discussion made by Justice Regalado, speaking for the
Court on the conflicting views on the manner of interpretation of the legal formalities required in the
execution of the attestation clause in wills.27 Uy Coque and Andrada are cited therein, along with several
other cases, as examples of the application of the rule of strict construction.28 However, the Code
Commission opted to recommend a more liberal construction through the "substantial compliance rule"
under Article 809. A cautionary note was struck though by Justice J.B.L. Reyes as to how Article 809
should be applied:
x x x The rule must be limited to disregarding those defects that can be supplied by an examination of the
will itself: whether all the pages are consecutively numbered; whether the signatures appear in each and
every page; whether the subscribing witnesses are three or the will was notarized. All these are facts that
the will itself can reveal, and defects or even omissions concerning them in the attestation clause can be
safely disregarded. But the total number of pages, and whether all persons required to sign did so in
the presence of each other must substantially appear in the attestation clause, being the only check
against perjury in the probate proceedings.29 (Emphasis supplied.)

The Court of Appeals did cite these comments by Justice J.B.L. Reyes in its assailed decision,
considering that the failure to state the number of pages of the will in the attestation clause is one of the
defects which cannot be simply disregarded. In Caneda itself, the Court refused to allow the probate of a
will whose attestation clause failed to state that the witnesses subscribed their respective signatures to the
will in the presence of the testator and of each other,30 the other omission cited by Justice J.B.L. Reyes
which to his estimation cannot be lightly disregarded.
Caneda suggested: "[I]t may thus be stated that the rule, as it now stands, is that omission which can be
supplied by an examination of the will itself, without the need of resorting to extrinsic evidence, will not
be fatal and, correspondingly, would not obstruct the allowance to probate of the will being assailed.
However, those omissions which cannot be supplied except by evidence aliunde would result in the
invalidation of the attestation clause and ultimately, of the will itself."31 Thus, a failure by the attestation
clause to state that the testator signed every page can be liberally construed, since that fact can be checked
by a visual examination; while a failure by the attestation clause to state that the witnesses signed in one
anothers presence should be considered a fatal flaw since the attestation is the only textual guarantee of
compliance.32
The failure of the attestation clause to state the number of pages on which the will was written remains a
fatal flaw, despite Article 809. The purpose of the law in requiring the clause to state the number of pages
on which the will is written is to safeguard against possible interpolation or omission of one or some of its
pages and to prevent any increase or decrease in the pages.33 The failure to state the number of pages
equates with the absence of an averment on the part of the instrumental witnesses as to how many pages
consisted the will, the execution of which they had ostensibly just witnessed and subscribed to.
Following Caneda, there is substantial compliance with this requirement if the will states elsewhere in it
how many pages it is comprised of, as was the situation inSingson and Taboada. However, in this case,
there could have been no substantial compliance with the requirements under Article 805 since there is no
statement in the attestation clause or anywhere in the will itself as to the number of pages which comprise
the will.
At the same time, Article 809 should not deviate from the need to comply with the formal requirements as
enumerated under Article 805. Whatever the inclinations of the members of the Code Commission in
incorporating Article 805, the fact remains that they saw fit to prescribe substantially the same formal
requisites as enumerated in Section 618 of the Code of Civil Procedure, convinced that these remained
effective safeguards against the forgery or intercalation of notarial wills.34 Compliance with these
requirements, however picayune in impression, affords the public a high degree of comfort that the
testator himself or herself had decided to convey property post mortem in the manner established in the
will.35 The transcendent legislative intent, even as expressed in the cited comments of the Code
Commission, is for the fruition of the testators incontestable desires, and not for the indulgent
admission of wills to probate.
The Court could thus end here and affirm the Court of Appeals. However, an examination of the will
itself reveals a couple of even more critical defects that should necessarily lead to its rejection.
For one, the attestation clause was not signed by the instrumental witnesses. While the signatures of
the instrumental witnesses appear on the left-hand margin of the will, they do not appear at the bottom of
the attestation clause which after all consists of their averments before the notary public.
Cagro v. Cagro36 is material on this point. As in this case, "the signatures of the three witnesses to the
will do not appear at the bottom of the attestation clause, although the page containing the same is signed
by the witnesses on the left-hand margin."37 While three (3) Justices38 considered the signature

requirement had been substantially complied with, a majority of six (6), speaking through Chief Justice
Paras, ruled that the attestation clause had not been duly signed, rendering the will fatally defective.
There is no question that the signatures of the three witnesses to the will do not appear at the bottom of
the attestation clause, although the page containing the same is signed by the witnesses on the left-hand
margin.
We are of the opinion that the position taken by the appellant is correct. The attestation clause is "a
memorandum of the facts attending the execution of the will" required by law to be made by the attesting
witnesses, and it must necessarily bear their signatures. An unsigned attestation clause cannot be
considered as an act of the witnesses, since the omission of their signatures at the bottom thereof
negatives their participation.
The petitioner and appellee contends that signatures of the three witnesses on the left-hand margin
conform substantially to the law and may be deemed as their signatures to the attestation clause. This is
untenable, because said signatures are in compliance with the legal mandate that the will be signed on the
left-hand margin of all its pages. If an attestation clause not signed by the three witnesses at the bottom
thereof, be admitted as sufficient, it would be easy to add such clause to a will on a subsequent occasion
and in the absence of the testator and any or all of the witnesses.39
The Court today reiterates the continued efficacy of Cagro. Article 805 particularly segregates the
requirement that the instrumental witnesses sign each page of the will, from the requisite that the will be
"attested and subscribed by [the instrumental witnesses]." The respective intents behind these two classes
of signature are distinct from each other. The signatures on the left-hand corner of every page signify,
among others, that the witnesses are aware that the page they are signing forms part of the will. On the
other hand, the signatures to the attestation clause establish that the witnesses are referring to the
statements contained in the attestation clause itself. Indeed, the attestation clause is separate and apart
from the disposition of the will. An unsigned attestation clause results in an unattested will. Even if the
instrumental witnesses signed the left-hand margin of the page containing the unsigned attestation clause,
such signatures cannot demonstrate these witnesses undertakings in the clause, since the signatures that
do appear on the page were directed towards a wholly different avowal.
The Court may be more charitably disposed had the witnesses in this case signed the attestation clause
itself, but not the left-hand margin of the page containing such clause. Without diminishing the value of
the instrumental witnesses signatures on each and every page, the fact must be noted that it is the
attestation clause which contains the utterances reduced into writing of the testamentary witnesses
themselves. It is the witnesses, and not the testator, who are required under Article 805 to state the
number of pages used upon which the will is written; the fact that the testator had signed the will and
every page thereof; and that they witnessed and signed the will and all the pages thereof in the presence of
the testator and of one another. The only proof in the will that the witnesses have stated these elemental
facts would be their signatures on the attestation clause.
Thus, the subject will cannot be considered to have been validly attested to by the instrumental witnesses,
as they failed to sign the attestation clause.
Yet, there is another fatal defect to the will on which the denial of this petition should also hinge. The
requirement under Article 806 that "every will must be acknowledged before a notary public by the
testator and the witnesses" has also not been complied with. The importance of this requirement is
highlighted by the fact that it had been segregated from the other requirements under Article 805 and
entrusted into a separate provision, Article 806. The non-observance of Article 806 in this case is equally

as critical as the other cited flaws in compliance with Article 805, and should be treated as of equivalent
import.
In lieu of an acknowledgment, the notary public, Petronio Y. Bautista, wrote "Nilagdaan ko at ninotario
ko ngayong10 ng Hunyo 10 (sic), 1981 dito sa Lungsod ng Maynila."40 By no manner of contemplation
can those words be construed as an acknowledgment. An acknowledgment is the act of one who has
executed a deed in going before some competent officer or court and declaring it to be his act or deed.41 It
involves an extra step undertaken whereby the signor actually declares to the notary that the executor of a
document has attested to the notary that the same is his/her own free act and deed.
It might be possible to construe the averment as a jurat, even though it does not hew to the usual language
thereof. A jurat is that part of an affidavit where the notary certifies that before him/her, the document
was subscribed and sworn to by the executor.42 Ordinarily, the language of the jurat should avow that the
document was subscribed and sworn before the notary public, while in this case, the notary public averred
that he himself "signed and notarized" the document. Possibly though, the word "ninotario" or
"notarized" encompasses the signing of and swearing in of the executors of the document, which in this
case would involve the decedent and the instrumental witnesses.
Yet even if we consider what was affixed by the notary public as a jurat, the will would nonetheless
remain invalid, as the express requirement of Article 806 is that the will be "acknowledged", and not
merely subscribed and sworn to. The will does not present any textual proof, much less one under oath,
that the decedent and the instrumental witnesses executed or signed the will as their own free act or deed.
The acknowledgment made in a will provides for another all-important legal safeguard against spurious
wills or those made beyond the free consent of the testator. An acknowledgement is not an empty
meaningless act.43 The acknowledgment coerces the testator and the instrumental witnesses to declare
before an officer of the law that they had executed and subscribed to the will as their own free act or deed.
Such declaration is under oath and under pain of perjury, thus allowing for the criminal prosecution of
persons who participate in the execution of spurious wills, or those executed without the free consent of
the testator. It also provides a further degree of assurance that the testator is of certain mindset in making
the testamentary dispositions to those persons he/she had designated in the will.
It may not have been said before, but we can assert the rule, self-evident as it is under Article 806. A
notarial will that is not acknowledged before a notary public by the testator and the witnesses is
fatally defective, even if it is subscribed and sworn to before a notary public.
There are two other requirements under Article 805 which were not fully satisfied by the will in question.
We need not discuss them at length, as they are no longer material to the
disposition of this case. The provision requires that the testator and the instrumental witnesses sign each
and every page of the will on the left margin, except the last; and that all the pages shall be numbered
correlatively in letters placed on the upper part of each page. In this case, the decedent, unlike the
witnesses, failed to sign both pages of the will on the left margin, her only signature appearing at the socalled "logical end"44 of the will on its first page. Also, the will itself is not numbered correlatively in
letters on each page, but instead numbered with Arabic numerals. There is a line of thought that has
disabused the notion that these two requirements be construed as mandatory.45 Taken in isolation, these
omissions, by themselves, may not be sufficient to deny probate to a will. Yet even as these omissions are
not decisive to the adjudication of this case, they need not be dwelt on, though indicative as they may be
of a general lack of due regard for the requirements under Article 805 by whoever executed the will.

All told, the string of mortal defects which the will in question suffers from makes the probate denial
inexorable.
WHEREFORE, the petition is DENIED. Costs against petitioner.
SO ORDERED.
DANTE O. TINGA
Associate Justice

C.A. No. 8075

March 25, 1946

TRINIDAD NEYRA, plaintiff-appellant,


vs.
ENCARNACION NEYRA, defendant-appellee.
Alejandro M. Panis for appellant.
Lucio Javillonar for appellee.
DE JOYA, J.:
On October 25, 1939, Trinidad Neyra filed a complaint against her sister, Encarnacion Neyra, in the Court
of First Instance of the City of Manila, for the recovery of one-half () of the property mentioned and
described therein, which had been left by their deceased father, Severo Neyra, and which had been
previously divided equally between the two extrajudicially, demanding at the same time one-half () of
the rents collected on the said property by the defendant Encarnacion Neyra. The defendant filed an
answer admitting that the property mentioned and described therein was community property, and at the
same time set up counterclaims amounting to over P1,000, for money spent, during the last illness of their
father, and for money loaned to the plaintiff.
After the trial of the case, the court found that the plaintiff was really entitled to one-half () of the said
property, adjudicating the same to her, but at the same time ordered said plaintiff to pay to the defendant
the sum of P727.77, plus interests, by virtue of said counterclaims.
Plaintiff Trinidad Neyra appealed from the said decision, to the Court of Appeals for Manila, alleging
several errors, attacking the execution and validity of said agreement; and on November 10, 1942, said
appeal was dismissed, pursuant to the to an agreement or compromise entered into by the parties, as
shown by the corresponding document, dated November 3, 1942, which was filed in the case the
following day, November 4, 1942.
In the meanwhile, Encarnacion Neyra, who had been sickly for about two years, unexpectedly died, on
November 4, 1942 at the age of 48, allegedly from heart attack, as a consequence of Addison's disease
from which, it was claimed, she had been suffering for sometime.
In view of the decision of the Court of Appeals, dated November 10, 1942, dismissing the appeal, by
virtue of said agreement or compromise, Atty. Lucio Javillonar, claiming to represent Encarnacion Neyra,
who had died since November 4, 1942, and other relatives of hers, filed a petition, dated November 23,

1942, asking for the reconsideration of said decision of the Court of Appeals, dismissing the appeal,
claiming that the alleged compromise or agreement, dated November 3, 1942, could not have been
understood by Encarnacion Neyra, as she was already then at the threshold of death, and that as a matter
of fact she died the following day; and that if it had been signed at all by said Encarnacion Neyra, her
thumbmark appearing on said document must have been affixed thereto by Trinidad Neyra's attorney,
against Encarnacion's will; and that the court had no more jurisdiction over the case, when the alleged
agreement was filed on November 4, 1942, at the instance of Trinidad Neyra, as Encarnacion was already
dead at the time.
The principal question to be decided, in connection with said petition for reconsideration, is whether or
not said compromise or agreement had been legally executed and signed by Encarnacion Neyra, on
November 3, 1942. Trinidad Neyra maintains the affirmative.
The voluminous evidence, testimonial and documentary, adduced by the parties, in this case, has fully
established the following facts:
That Severo Nayra died intestate in the City of Manila, on May 6, 1938, leaving certain properties and
two children, by his first marriage, named Encarnacion Neyra and Trinidad Neyra, and other children by
his second marriage; That after the death of Severo Neyra, the two sisters, Encarnacion Neyra and
Trinidad Neyra, had serious misunderstandings, in connection with the properties left by their deceased
father, and so serious were their dissensions that, after March 31, 1939, they had two litigations in the
Court of First Instance of Manila, concerning said properties. In the first case, filed in March 31, 1939,
Trinidad Neyra and others demanded by Encarnacion Neyra and others the annulment of the sale of the
property located at No. 366 Raon Street, Manila which was finally decided in favor of the defendants, in
the court of first instance, and in the Court of Appeals, on December 21, 1943 (G.R. No. 8162); and the
second is the instance case.
That Encarnacion Neyra, who had remained single, and who had no longer any ascendants, executed a
will on September 14, 1939, marked Exhibit 16, disposing of her properties in favor of the "Congregacion
de Religiosas de la Virgen Maria" and her other relatives, named Teodora Neyra, Pilar de Guzman and
Maria Jacobo Vda. de Blanco, making no provision whatsoever in said will, in favor of her only sister of
the whole blood, Trinidad Neyra, who had become her bitter enemy; that when the said will was brought
to the attention of the authorities of said Congregation, after due deliberation and consideration, said
religious organization declined the bounty offered by Encarnacion Neyra, and said decision of the
Congregation was duly communicated to her; that in order to overcome the difficulties encountered by
said religious organization in not accepting the generosity of Encarnacion Neyra, the latter decided to
make a new will, and for that purpose, about one week before her death, sent for Atty. Ricardo Sikat, and
gave him instructions for the preparation of a new will; that Atty. Sikat, instead of preparing a new will,
merely prepared a draft of a codicil, amending said will, dated September 14, 1939, again naming said
religious organization, among others as beneficiary, and said draft of a codicil was also forwarded to the
authorities of religious organization, for their consideration and acceptance; but it was also rejected.
In the meanwhile, Encarnacion Neyra had become seriously ill, suffering from Addison's disease, and on
October 31, 1942, she sent for her religious adviser and confessor, Mons. Vicente Fernandez of the
Quiapo Church to make confession, after which she requested that holy mass be celebrated in her house at
No. 366 Raon Street, City of Manila, so that she might take holy communion; that Mons. Fernandez
caused the necessary arrangements to be made, and, as a matter of fact, on November 1, 1942, holy mass
was solemnized in her house by Father Teodoro Garcia, also of the Quiapo Church, on which occasion,
Encarnacion Neyra, who remained in bed, took holy communion; that after the mass, Father Garcia talked
to Encarnacion Neyra and advised reconciliation between the two sisters, Encarnacion and Trinidad

Neyra. Encarnacion accepted said advise and, at about noon of the same day (November 1, 1942), sent
Eustaquio Mendoza to fetch her sister Trinidad, who came at about 2:30 that same afternoon; that the two
sisters greeted each other in most affectionate manner, and became reconciled and two had a long and
cordial conversation, in the course of which they also talked about the properties left by their father and
their litigations which had reached the Court of Appeals for the City of Manila, the instant case being the
second, and they agreed to have the latter dismissed, on the condition that the property involved therein
should be given exclusively to Trinidad Neyra, that the latter should waive her share in the rents of said
property collected by Encarnacion, and the Trinidad had no more indebtedness to Encarnacion. They also
agreed to send for Atty. Alejandro M. Panis, to prepare the necessary document embodying the said
agreement, but Attorney Panis could come only in the afternoon of the following day, November 2, 1942,
when Encarnacion gave him instructions for the preparation of the document embodying their agreement,
and other instructions for the preparation of her last will and testament; that Attorney Panis prepared said
document of compromise as well as the new will and testament, naming Trinidad Neyra and Eustaquio
Mendoza beneficiaries therein, pursuant to Encarnacion's express instructions, and the two documents
were prepared, in duplicate, and were ready for signature, since the morning of November 3, 1942; that in
the afternoon of that day, of compromise and last will and testament to Encarnacion Neyra, slowly and in
a loud voice, in the presence of Father Teodoro Garcia, Dr. Moises B. Abad, Dr. Eladio Aldecoa, Trinidad
Neyra, and others, after which he asked her if their terms were in accordance with her wishes, or if she
wanted any change made in said documents; that Encarnacion Neyra did not suggest any change, and
asked for the pad and the two documents, and, with the help of a son of Trinidad, placed her thumbmark
at the foot of each one of the two documents, in duplicate, on her bed in the sala, in the presence of
attesting witnesses, Dr. Moises B. Abad, Dr. Eladio R. Aldecoa and Atty. Alejandro M. Panis, after which
said witnesses signed at the foot of the will, in the presence of Encarnacion Neyra, and of each other. The
agreement was also signed by Trinidad Neyra, as party, and by Dr. M. B. Abad and Eustaquio Mendoza, a
protege, as witnesses.
Father Teodoro Garcia was also present at the signing of the two documents, at the request of
Encarnacion Neyra.
The foregoing facts have been established by the witnesses presented by Trinidad Neyra, who are all
trustworthy men, and who had absolutely no interest in the final outcome of this case. Two of them are
ministers of the Gospel, while three of the attesting witnesses are professional men of irreproachable
character, who had known and seen and actually talked to the testatrix.
Petitioner Teodora Neyra, half sister of Encarnacion, and her young daughter Ceferina de la Cruz, and
Presentacion Blanco, daughter of petitioner Maria Jacobo Vda. de Blanco, substantially corroborated the
testimony of the witnesses presented by Trinidad Neyra, with reference to the signing of documents, in
the bedroom of Encarnacion Neyra, in the afternoon of November 3, 1942.
Teodora Neyra, Presentacion Blanco and Ceferina de la Cruz testified, however, that when the
thumbmark of Encarnacion Neyra was affixed to the agreement in question, dated November 3, 1942, she
was sleeping on her bed in the sala; and that the attesting witnesses were not present, as they were in
the caida.
But Ceferina de la Cruz also stated that the attesting witnesses signed the documents thumbmarked by
Encarnacion Neyra, in the sala near her bed, thus contradicting herself and Teodora Neyra and
Presentacion Blanco.

Strange to say, Teodora Neyra, Presentacion Blanco and Ceferina de la Cruz also testified that
Encarnacion Neyra's, thumbmark was affixed to the will, only in the morning of November 4, 1942, by
Trinidad Neyra and one Ildefonso del Barrio, when Encarnacion was already dead.
The testimony of Dr. Dionisio Parulan, alleged medical expert, as to the nature of effects of Addison's
disease, is absolutely unreliable. He had never seen or talked to the testatrix Encarnacion Neyra.
According to medical authorities, persons suffering from Addison's disease often live as long as ten (10)
years, while others die after a few weeks only, and that as the disease progresses, asthenia sets in, and
from 80 per cent to 90 per cent of the patients develop tuberculosis, and complications of the heart also
appear. (Cecil, Textbook of Medicine, 3d ed., 1935, pp. 1250-1253; McCrae, Osler's Modern Medicine,
3d ed., Vol. V, pp. 272-279.)
And it has been conclusively shown that Encarnacion Neyra died on November 4, 1942, due to a heart
attack, at the age of 48, after an illness of about two (2) years.
In connection with mental capacity, in several cases, this court has considered the testimony of witnesses,
who had known and talked to the testators, more trustworthy than the testimony of the alleged medical
experts.
Insomnia, in spite of the testimony of two doctors, who testified for the opponents to the probate of a will,
to the effect that it tended to destroy mental capacity, was held not to effect the full possession of mental
faculties deemed necessary and sufficient for its execution. (Caguioa vs. Calderon, 20 Phil., 400.) The
testatrix was held to have been compos mentis, in spite of the physician's testimony to the contrary, to the
effect that she was very weak, being in the third or last stage of tuberculosis. (Yap Tua vs. Yap Ca Kuan
and Yap Ca Llu, 27 Phil., 579.) The testimony of the attending physician that the deceased was suffering
from diabetes and had been in a comatose condition for several days, prior to his death, was held not
sufficient to establish testamentary incapacity, in view of the positive statement of several credible
witnesses that he was conscious and able to understand what was said to him and to communicate his
desires. (Samson vs. Corrales Tan Quintin, 44 Phil., 573.) Where the mind of the testator is in perfectly
sound condition, neither old age, nor ill health, nor the fact that somebody had to guide his hand in order
that he might sign, is sufficient to invalidate his will (Amata and Almojuela vs. Tablizo, 48 Phil., 485.)
Where it appears that a few hours and also a few days after the execution of the will, the testator
intelligently and intelligibly conversed with other persons, although lying down and unable to move or
stand up unassisted, but could still effect the sale of property belonging to him, these circumstances show
that the testator was in a perfectly sound mental condition at the time of the execution of the will. (Amata
and Almojuela vs. Tablizo, 48 Phil., 485.)
Presentacion Blanco, in the course of her cross-examination, frankly admitted that, in the morning and
also at about 6 o'clock in he afternoon of November 3, 1942, Encarnacion Neyra talked to her that they
understood each other clearly, thus showing that the testatrix was really of sound mind, at the time of
signing and execution of the agreement and will in question.
It may, therefore, be reasonably concluded that the mental faculties of persons suffering from Addison's
disease, like the testatrix in this case, remain unimpaired, partly due to the fact that, on account of the
sleep they enjoy, they necessarily receive the benefit of physical and mental rest. And that like patients
suffering from tuberculosis, insomnia or diabetes, they preserve their mental faculties until the moments
of their death.

Judging by the authorities above cited, the logical conclusion is that Encarnacion Neyra was of sound
mind and possessed the necessary testamentary and mental capacity, at the time of the execution of the
agreement and will, dated November 3, 1942.
The contention that the attesting witnesses were not present, at the time Encarnacion Neyra thumbmarked
the agreement and will in question, on her bed, in the sala of the house, as they were allegedly in
the caida, is untenable. It has been fully shown that said witnesses were present, at the time of the signing
and execution of the agreement and will in question, in the sala, where the testatrix was lying on her bed.
The true test is not whether they actually saw each other at the time of the signing of the documents, but
whether they might have seen each other sign, had they chosen to do so; and the attesting witnesses
actually saw it all in this case. (Jaboneta vs. Gustilo, 5 Phil., 541.) And the thumbmark placed by the
testatrix on the agreement and will in question is equivalent to her signature. (Yap Tua vs. Yap Ca Kuan
and Yap Ca Llu, 27 Phil., 579.)
Teodora Neyra and her principal witnesses are all interested parties, as they are children of legatees
named in the will, dated September 14, 1939, but eliminated from the will, dated November 3, 1942.
Furthermore, the testimony of Teodora Neyra and her witnesses, to the effect that there could have been
no reconciliation between the two sisters, and that the thumbmark of Encarnacion Neyra was affixed to
the documents embodying the agreement, while she was sleeping, on November 3, 1942, in their
presence; and that her thumbmark was affixed to the will in question, when she was already dead, in the
morning of November 4, 1942, within their view, is absolutely devoid of any semblance of truth. Said
testimony is contrary to common sense. It violates all sense of proportion. Teodora Neyra and her
witnesses could not have told the truth; they have testified to deliberate falsefoods; and they are,
therefore, absolutely unworthy of belief. And to the evidence of the petitioners is completely applicable
the legal aphorism falsus in uno, falsus in omnibus. (Gonzales vs.Mauricio, 53 Phil., 728, 735.)
To show the alleged improbability of reconciliation, and the execution of the two documents, dated
November 3, 1942, petitioners have erroneously placed great emphasis on the fact that, up to October 31,
1942, the two sisters Encarnacion and Trinidad Neyra were bitter enemies. They were banking evidently
on the common belief that the hatred of relatives is the most violent. Terrible indeed are the feuds of
relatives and difficult the reconciliation; and yet not impossible. They had forgotten that Encarnacion
Neyra was a religious woman instructed in the ancient virtues of the Christian faith, and hope and charity,
and that to forgive is a divine attribute. They had also forgotten that there could be no more sublime love
than that embalmed in tears, as in the case of a reconciliation.
It was most natural that there should have been reconciliation between the two sisters, Encarnacion and
Trinidad Neyra, as the latter is the nearest relative of the former, her only sister of the whole blood. The
approach of imminent death must have evoked in her the tenderest recollections of family life. And
believing perhaps that her little triumphs had not always brought her happiness, and that she had always
been just to her sister, who had been demanding insistently what was her due, Encarnacion finally decided
upon reconciliation, as she did not want to go to her eternal rest, with hatred in her heart or wrath upon
her head. It was, therefore, most logical that Encarnacion should make Trinidad the benificiary of her
generosity, under her last will and testament, and end all her troubles with her, by executing said
agreement, and thus depart in perfect peace from the scenes of her earthly labors.
It having been shown that the said compromise or agreement had been legally signed and executed by
Encarnacion Neyra on November 3, 1942, in the presence of credible and trustworthy witnesses, and that
she was compos mentis and possessed the necessary testamentary and mental capacity of the time; the
petition for the reconsideration filed by Atty. Lucio Javillonar, on November 23, 1942, on behalf of a

client, Encarnacion Neyra, who had been dead since November 4, 1942, and some of her relatives, who
have appeared, in accordance with the provisions of section 17 of Rule 3 of the Rules of Court, is
hereby denied; and the decision of the Court of Appeals for Manila, dated November 10, 1942, dismissing
the appeal, is hereby re-affirmed, without costs. So ordered.
Ozaeta, Perfecto, Hilado, and Bengzon, JJ., concur.

FIRST DIVISION
[G.R. No. 174144. April 17, 2007.]
BELLA A. GUERRERO, petitioner, vs. RESURRECCION A. BIHIS, respondent.
D E C I S I O N CORONA, J p:
The Scriptures tell the story of the brothers Jacob and Esau 1 , siblings who fought bitterly over the
inheritance of their father Isaac's estate. Jurisprudence is also replete with cases involving acrimonious
conflicts between brothers and sisters over successional rights. This case is no exception.
On February 19, 1994, Felisa Tamio de Buenaventura, mother of petitioner Bella A. Guerrero and
respondent Resurreccion A. Bihis, died at the Metropolitan Hospital in Tondo, Manila.
On May 24, 1994, petitioner filed a petition for the probate of the last will and testament of the decedent
in Branch 95 2 of the Regional Trial Court of Quezon City where the case was docketed as Sp. Proc. No.
Q-94-20661.
The petition alleged the following: petitioner was named as executrix in the decedent's will and she was
legally qualified to act as such; the decedent was a citizen of the Philippines at the time of her death; at
the time of the execution of the will, the testatrix was 79 years old, of sound and disposing mind, not
acting under duress, fraud or undue influence and was capacitated to dispose of her estate by will.
Respondent opposed her elder sister's petition on the following grounds: the will was not executed and
attested as required by law; its attestation clause and acknowledgment did not comply with the
requirements of the law; the signature of the testatrix was procured by fraud and petitioner and her
children procured the will through undue and improper pressure and influence.
In an order dated November 9, 1994, the trial court appointed petitioner as special administratrix of the
decedent's estate. Respondent opposed petitioner's appointment but subsequently withdrew her
opposition. Petitioner took her oath as temporary special administratrix and letters of special
administration were issued to her.
On January 17, 2000, after petitioner presented her evidence, respondent filed a demurrer thereto alleging
that petitioner's evidence failed to establish that the decedent's will complied with Articles 804 and 805 of
the Civil Code.
In a resolution dated July 6, 2001, the trial court denied the probate of the will ruling that Article 806 of
the Civil Code was not complied with because the will was "acknowledged" by the testatrix and the
witnesses at the testatrix's residence at No. 40 Kanlaon Street, Quezon City before Atty. Macario O.
Directo who was a commissioned notary public for and in Caloocan City. The dispositive portion of the
resolution read:
WHEREFORE, in view of the foregoing, the Court finds, and so declares that it cannot admit the last will
and testament of the late Felisa Tamio de Buenaventura to probate for the reasons hereinabove discussed
and also in accordance with Article 839 [of the Civil Code] which provides that if the formalities required
by law have not been complied with, the will shall be disallowed. In view thereof, the Court shall
henceforth proceed with intestate succession in regard to the estate of the deceased Felisa Tamio de
Buenaventura in accordance with Article 960 of the [Civil Code], to wit: "Art. 960. Legal or intestate
succession takes place: (1) If a person dies without a will, or with a void will, or one which has
subsequently lost its validity, . . . ." SO ORDERED, 3

Petitioner elevated the case to the Court of Appeals but the appellate court dismissed the appeal and
affirmed the resolution of the trial court. 4 Thus, this petition. 5 Petitioner admits that the will was
acknowledged by the testatrix and the witnesses at the testatrix's residence in Quezon City before Atty.
Directo and that, at that time, Atty. Directo was a commissioned notary public for and in Caloocan City.
She, however, asserts that the fact that the notary public was acting outside his territorial jurisdiction did
not affect the validity of the notarial will.
Did the will "acknowledged" by the testatrix and the instrumental witnesses before a notary public acting
outside the place of his commission satisfy the requirement under Article 806 of the Civil Code? It did
not. Article 806 of the Civil Code provides:
ART. 806. Every will must be acknowledged before a notary public by the testator and the witnesses. The
notary public shall not be required to retain a copy of the will, or file another with the office of the Clerk
of Court.
One of the formalities required by law in connection with the execution of a notarial will is that it must
be acknowledged before a notary public by the testator and the witnesses. 6 This formal requirement is
one of the indispensable requisites for the validity of a will. 7 In other words, a notarial will that is not
acknowledged before a notary public by the testator and the instrumental witnesses is void and cannot be
accepted for probate. cDICaS
An acknowledgment is the act of one who has executed a deed in going before some competent officer
and declaring it to be his act or deed. 8 In the case of a notarial will, that competent officer is the notary
public.
The acknowledgment of a notarial will coerces the testator and the instrumental witnesses to declare
before an officer of the law, the notary public, that they executed and subscribed to the will as their own
free act or deed. 9 Such declaration is under oath and under pain of perjury, thus paving the way for the
criminal prosecution of persons who participate in the execution of spurious wills, or those executed
without the free consent of the testator. 10 It also provides a further degree of assurance that the testator is
of a certain mindset in making the testamentary dispositions to the persons instituted as heirs or
designated as devisees or legatees in the will. 11
Acknowledgment can only be made before a competent officer, that is, a lawyer duly commissioned as a
notary public. In this connection, the relevant provisions of the Notarial Law provide:
SECTION 237.
Form of commission for notary public. The appointment of a notary public shall be in writing, signed
by the judge, and substantially in the following form:
GOVERNMENT OF THE REPUBLIC OF THE PHILIPPINES
PROVINCE OF ____________
This is to certify that ______, of the municipality of ______ in said province, was on the ____ day of
______, anno Domini nineteen hundred and _____, appointed by me a notary public, within and for the
said province, for the term ending on the first day of January, anno Domini nineteen hundred and _____.
__________________
Judge of the Court of First Instance 12 of said Province xxx xxx xxx
SECTION 240. Territorial jurisdiction. The jurisdiction of a notary public in a province shall be coextensive with the province. The jurisdiction of a notary public in the City of Manila shall be co-extensive
with said city. No notary shall possess authority to do any notarial act beyond the limits of his
jurisdiction. (emphases supplied)
A notary public's commission is the grant of authority in his favor to perform notarial acts. 13 It is issued
"within and for" a particular territorial jurisdiction and the notary public's authority is co-extensive with it.
In other words, a notary public is authorized to perform notarial acts, including the taking of
acknowledgments, within that territorial jurisdiction only. Outside the place of his commission, he is
bereft of power to perform any notarial act; he is not a notary public. Any notarial act outside the limits of
his jurisdiction has no force and effect. As this Court categorically pronounced in Tecson v. Tecson: 14
An acknowledgment taken outside the territorial limits of the officer's jurisdiction is void as if the person
taking it were wholly without official character. (emphasis supplied) AECDHS

Since Atty. Directo was not a commissioned notary public for and in Quezon City, he lacked the authority
to take the acknowledgment of the testatrix and the instrumental witnesses. In the same vein, the testatrix
and her witnesses could not have validly acknowledged the will before him. Thus, Felisa Tamio de
Buenaventura's last will and testament was, in effect, not acknowledged as required by law.
Moreover, Article 5 of the Civil Code provides: ART. 5. Acts executed against the provisions of
mandatory or prohibitory laws shall be void, except when the law itself authorizes their validity.
The violation of a mandatory or a prohibitory statute renders the act illegal and void unless the law itself
declares its continuing validity. Here, mandatory and prohibitory statutes were transgressed in the
execution of the alleged "acknowledgment." The compulsory language of Article 806 of the Civil Code
was not complied with and the interdiction of Article 240 of the Notarial Law was breached. Ineluctably,
the acts of the testatrix, her witnesses and Atty. Directo were all completely void.
The Court cannot turn a blind eye to Atty. Directo's participation in the preparation, execution and
unlawful "acknowledgment" of Felisa Tamio de Buenaventura's will. Had he exercised his notarial
commission properly, the intent of the law to effectuate the decedent's final statements 15 as expressed in
her will would not have come to naught. 16 Hence, Atty. Directo should show cause why he should not
be administratively sanctioned as a member of the bar and as an officer of the court.
WHEREFORE, the petition is hereby DENIED. Costs against petitioner.
________________________________________________________________________
In the case of Leynez vs. Leynez (40 Off. Gaz., 3rd Supplement, 51, 52, No. 7, October 18, 1939; 68 Phil.,
745), the attestation clause reads as follows:
Suscrito y declarado por el testador Valerio Leynez, como su ultima voluntad y testamento en
presencia de todos y cada uno de nosotros, y a ruego de dicho testador, firmamos el presente cada
uno en presencia de los otros, o de los demas y de la del mismo testsador, Valerio Leynez. El
testamento consta de dos (2) paginas solamente.
The objection was that the attestation clause did not state that the testator and the witnesses signed each
and every page of the will. This fact , however, appears in the will itself. It is clear, therefore, that in case
of the will complied with all the requisites for its due execution. In the instant case, essential words were
omitted.
___
Republic of the Philippines SUPREME COURT Manila
EN BANC G.R. No. L-46097 October 18, 1939
TEOFILA ADEVA VIUDA DE LEYNEZ, petitioner, vs. IGNACIO LEYNEZ, respondent.
Conrado V. Sanchez and Ambrosio Padilla for petitioner.
Ilao and Enriquez for respondent.
LAUREL, J.: This is a petition for a writ of certiorari to review the decision of the Court of Appeals
affirming the decision of the Court of First Instance of Mindoro denying probate of the will of the

deceased Valerio Leynez, on the ground that its attestation clause does not conform to the requirements
section 618, as amended, of the Code of Civil Procedure.
The attestation clause of the will is worded as follows:
Suscrito y declarado por el testador Valerio Leynez, como su ultima voluntad y testamento en presencia
de todos y cada uno de nosotros, y a ruego de dicho testador, firmamos el presente cada uno en presencia
de los otros, o de los demas y de la del mismo testador Valerio Leynez . El testamento consta de los (2)
paginas solamente.
The question presented is, under section 618, as amended, of the Code of Civil Procedure, is this
attestation clause legally sufficient? The pertinent portion of this section of the Code is as follows:
. . . the attestation shall state the number of sheets or pages used, upon which the will is written, and the
fact that the testator signed the will and every page thereof, or caused some other person to write his
name, under his express direction, in the presence of three witnesses, an the later witnessed and signed the
will and all pages thereof in the presence of the testator and of each other.
The alleged defect in the attestation clause of the controverted will is that it fails to state that the testator
and the three witnesses signed each and every page of the will in the manner prescribed by law, because it
merely states "firmanos el presente cada uno en presencia de los otros, o de los demas y de la del mismo
testador Valerio Leynez." In deciding this question the Court of Appeals, however, ruled:
A la luz de las jurisprudencias arriba citadas en la clausada de atestiguamiento discutida en el asunto de
autos no encontramos un cumplimiento sustantial del requisito exigido por la ley, de que en ella se haga
constar que el testador y los testigos han firmado unos en presencia de otros, todas y cada una de las
paginas usadas del testamento, requisito que no se puede establecer por medio de su prueba aliunde.
Against this conclusion of the Court of Appeals, petitioner puts forward the contention that it has decided
a question of substance in a way not probably in accord with the law and the applicable decisions of this
court (Rule 47, paragraph e [1] of Supreme Court.) The rule of liberal construction of the applicable law
should, petitioner avers, be held to apply in the case at bar, and in support of her content on she invokes a
long array of cases (Abangan vs .Abangan, 40 Phil., 476; Avera vs. Garcia and Rodriguez, 42 Phil., 145;
Aldaba vs. Roque, 43 Phil., 378; Unson vs .Abella, 43 Phil., 494; Fernandez vs. Vergel de Dios, 46 Phil.,
922; Nayve vs. Mojal, 47 Phil., 152; De Gala vs .Gonzalez, 53 Phil., 104; Rey vs. Cartagena, 56 Phil.,
282; Dichoso de Ticson vs. De Gorostiza, 57 Phil., 437; Sebastian vs. Paganiban, 59 Phil., 653; De
Guzman vs. Celestino, G.R. No. 35273, April 25, 1932; Policarpio vs. Baltazar, G.R. No. 36349,
November 14, 1932; Malate vs. Olea, G.R. No. 36154, December 16, 1932; In re Estate of Jennings,
1933, G.R. No. 38758). To this line of cases those of Rodriguez vs. Yap, G.R. No. 45924, May 18, 1939,
and Grey vs. Fabie, G.R. No. 45160, May 23, 1939, may perhaps be added. Respondent, on the other
hand, equally invokes a number of cases wherein, he contends, the rule of strict construction was made to
prevail. (Uy Coque vs. Navas L. Sioca, 43 Phil., 405; In re Estate of Neuark, 46 Phil., 841; Sao
vs.Quintana, 48 Phil., 506; Gumban vs. Gorecho, 50 Phil., 30; Quinto vs. Morata, 54 Phil., 481;
Rodriguez vs. Alcala, 55 Phil., 150.)
This Court has already taken notice of these different views within, in Dichoso de Ticson vs. De
Gorostiza (57 Phil., 437, 439-440), it frankly made the following observation : "The truth is that there
have been, noticeable in the Philippines two divergent tendencies in the law of wills the one being
planted on strict construction and the other on liberal construction. A late example of the former views
may be found in the decision in Rodriguez vs. Alcala ([1930], 55 Phil., 150), sanctioning a literal

enforcement of the law. The basic case in the other direction, predicated on reason, is Abangan vs.
Abangan ([1919], 40 Phil., 476), oft-cited approvingly in later decisions." It is fairness to recognize the
existence of opposing currents of legal thought, a situation which perhaps has brought about a certain
degree of confusion in this field. It is also fairness to avow, however, that a more careful examination of
the cases will show that, while the two tendencies mentioned in easily discernible, the conflict in many
cases is more apparent than real, and the variance, if at all, in the application of the principles involved
was due in some instances to the marked differentiation of facts and the consequent personal or collective
criteria in particular cases.
We have taken pains to examine the numerous cases relied upon by the petitioner and those relied upon
by the respondent, and while we do not deem it necessary to make a detailed comparison between them,
we find no difficulty in selecting what we consider is the reasonable rule to apply in this case at bar. It is,
of course, not possible to lay down a general rule, rigid and inflexible, which would be applicable to all
cases. More than anything else, the facts and circumstances of record are to be considered in the
application of any given rule. If the surrounding circumstances point to a regular execution of the will,
and the instrument appears to have been executed substantially in accordance with the requirements of the
law, the inclination should, in the absence of any suggestion of bad faith, forgery or fraud, lean towards
its admission to probate, although the document may suffer from some imperfection of language, or other
non-essential defect. This, in our opinion, is the situation in the present case, and we, therefore, hold that
the requirement that the attestation clause, among other things, shall state "that the testator signed the will
and every page thereof in the presence of three witnesses, and that the witnesses signed the will in the
presence of the testator and of each other," is sufficiently complied with, it appearing that the testator and
the witnesses signed each and every page of the will according to the stipulation of the parties. (Record on
Appeal, stipulation, pp. 10, 14, 15); and this fact being shown in the will itself, and there being,
furthermore, no question raised as to the authenticity of the signature of the testator and the witnesses.
An attestation clause is made for the purpose of preserving, in permanent form, a record of the facts
attending the execution of the will, so that in case of failure of the memory of the subscribing witnesses,
or other casualty, they may still be proved. (Thompson on Wills, 2 ed., sec. 132.) A will, therefore, should
not be rejected where its attestation clause serves the purpose of the law. The law-making body, in
recognition of the dangers to which testamentary dispositions are apt to be subject in the hands of
unscrupulous individuals, has surrounded the execution of the wills with every solemnity deemed
necessary to safeguard it. This purpose was indicated when our legislature provided for the exclusion of
evidence aliunde to prove the execution of the will. We should not, however, attribute the prohibition as
indicative of a desire to impose unreasonable restraint or beyond what reason and justice permit. It could
not have been the intention of the legislature in providing for the essential safeguards in the execution of a
will to shackle the very right of testamentary disposition which the law recognizes and holds sacred. The
pronouncement of this Court in Abangan vs. Abangan (40 Phil., 476, 479), expresses the sound rule to
which we have recently adhered in principle. (Rodriguez vs. Yap, G.R. No. 45924, promulgated May 18,
1939; and Grey vs. Fabie, G.R. No. 45160, promulgated May 23, 1939):
The object of the solemnities surrounding the execution of wills is to close the door against bad faith and
fraud, to avoid substitution of wills and testaments and to guaranty their truth and authenticity. Therefore
the laws on this subject should be interpreted in such a way as to attain these primordial ends. But, on the
other hand, also one must not lose sight of the fact that it is not the object of the law to restrain and curtail
the exercise of the right to make a will. So when an interpretation already given assures such ends, any
other interpretation whatsoever, that adds nothing but demands more requisites entirely unnecessary,
useless and frustrative of the testator's last will, must be disregarded.

It follows that the writ of certiorari should be, as it is hereby, granted and the judgment of the Court of
Appeals reversed, with the result that the controverted will, Exhibit A, of the deceased Valerio Leynez,
shall be admitted to probate. So ordered, with costs against the respondent-appellee.
So ordered.
Avancea, C.J., Villa-Real, Imperial, Diaz, and Concepcion, JJ., concur.
G.R. No. L-7188

August 9, 1954

In re: Will and Testament of the deceased REVEREND SANCHO ABADIA.


SEVERINA A. VDA. DE ENRIQUEZ, ET AL., petitioners-appellees,
vs.
MIGUEL ABADIA, ET AL., oppositors-appellants.
Manuel A. Zosa, Luis B. Ladonga, Mariano A. Zosa and B. G. Advincula for appellants.
C. de la Victoria for appellees.
MONTEMAYOR, J.:
On September 6, 1923, Father Sancho Abadia, parish priest of Talisay, Cebu, executed a document
purporting to be his Last Will and Testament now marked Exhibit "A". Resident of the City of Cebu, he
died on January 14, 1943, in the municipality of Aloguinsan, Cebu, where he was an evacuee. He left
properties estimated at P8,000 in value. On October 2, 1946, one Andres Enriquez, one of the legatees in
Exhibit "A", filed a petition for its probate in the Court of First Instance of Cebu. Some cousins and
nephews who would inherit the estate of the deceased if he left no will, filed opposition.
During the hearing one of the attesting witnesses, the other two being dead, testified without contradiction
that in his presence and in the presence of his co-witnesses, Father Sancho wrote out in longhand Exhibit
"A" in Spanish which the testator spoke and understood; that he (testator) signed on he left hand margin
of the front page of each of the three folios or sheets of which the document is composed, and numbered
the same with Arabic numerals, and finally signed his name at the end of his writing at the last page, all
this, in the presence of the three attesting witnesses after telling that it was his last will and that the said
three witnesses signed their names on the last page after the attestation clause in his presence and in the
presence of each other. The oppositors did not submit any evidence.
The learned trial court found and declared Exhibit "A" to be a holographic will; that it was in the
handwriting of the testator and that although at the time it was executed and at the time of the testator's
death, holographic wills were not permitted by law still, because at the time of the hearing and when the
case was to be decided the new Civil Code was already in force, which Code permitted the execution of
holographic wills, under a liberal view, and to carry out the intention of the testator which according to
the trial court is the controlling factor and may override any defect in form, said trial court by order dated
January 24, 1952, admitted to probate Exhibit "A", as the Last Will and Testament of Father Sancho
Abadia. The oppositors are appealing from that decision; and because only questions of law are involved
in the appeal, the case was certified to us by the Court of Appeals.
The new Civil Code (Republic Act No. 386) under article 810 thereof provides that a person may execute
a holographic will which must be entirely written, dated and signed by the testator himself and need not
be witnessed. It is a fact, however, that at the time that Exhibit "A" was executed in 1923 and at the time

that Father Abadia died in 1943, holographic wills were not permitted, and the law at the time imposed
certain requirements for the execution of wills, such as numbering correlatively each page (not folio or
sheet) in letters and signing on the left hand margin by the testator and by the three attesting witnesses,
requirements which were not complied with in Exhibit "A" because the back pages of the first two folios
of the will were not signed by any one, not even by the testator and were not numbered, and as to the
three front pages, they were signed only by the testator.
Interpreting and applying this requirement this Court in the case of In re Estate of Saguinsin, 41 Phil.,
875, 879, referring to the failure of the testator and his witnesses to sign on the left hand margin of every
page, said:
. . . . This defect is radical and totally vitiates the testament. It is not enough that the signatures
guaranteeing authenticity should appear upon two folios or leaves; three pages having been
written on, the authenticity of all three of them should be guaranteed by the signature of the
alleged testatrix and her witnesses.
And in the case of Aspe vs. Prieto, 46 Phil., 700, referring to the same requirement, this Court declared:
From an examination of the document in question, it appears that the left margins of the six pages
of the document are signed only by Ventura Prieto. The noncompliance with section 2 of Act No.
2645 by the attesting witnesses who omitted to sign with the testator at the left margin of each of
the five pages of the document alleged to be the will of Ventura Prieto, is a fatal defect that
constitutes an obstacle to its probate.
What is the law to apply to the probate of Exh. "A"? May we apply the provisions of the new Civil Code
which not allows holographic wills, like Exhibit "A" which provisions were invoked by the appelleepetitioner and applied by the lower court? But article 795 of this same new Civil Code expressly provides:
"The validity of a will as to its form depends upon the observance of the law in force at the time it is
made." The above provision is but an expression or statement of the weight of authority to the affect that
the validity of a will is to be judged not by the law enforce at the time of the testator's death or at the time
the supposed will is presented in court for probate or when the petition is decided by the court but at the
time the instrument was executed. One reason in support of the rule is that although the will operates
upon and after the death of the testator, the wishes of the testator about the disposition of his estate among
his heirs and among the legatees is given solemn expression at the time the will is executed, and in reality,
the legacy or bequest then becomes a completed act. This ruling has been laid down by this court in the
case of In re Will of Riosa, 39 Phil., 23. It is a wholesome doctrine and should be followed.
Of course, there is the view that the intention of the testator should be the ruling and controlling factor
and that all adequate remedies and interpretations should be resorted to in order to carry out said
intention, and that when statutes passed after the execution of the will and after the death of the testator
lessen the formalities required by law for the execution of wills, said subsequent statutes should be
applied so as to validate wills defectively executed according to the law in force at the time of execution.
However, we should not forget that from the day of the death of the testator, if he leaves a will, the title of
the legatees and devisees under it becomes a vested right, protected under the due process clause of the
constitution against a subsequent change in the statute adding new legal requirements of execution of
wills which would invalidate such a will. By parity of reasoning, when one executes a will which is
invalid for failure to observe and follow the legal requirements at the time of its execution then upon his
death he should be regarded and declared as having died intestate, and his heirs will then inherit by
intestate succession, and no subsequent law with more liberal requirements or which dispenses with such
requirements as to execution should be allowed to validate a defective will and thereby divest the heirs of

their vested rights in the estate by intestate succession. The general rule is that the Legislature can not
validate void wills (57 Am. Jur., Wills, Sec. 231, pp. 192-193).
In view of the foregoing, the order appealed from is reversed, and Exhibit "A" is denied probate. With
costs.
Paras, C.J., Pablo, Bengzon, Padilla, Reyes, A., Jugo, Bautista Angelo, Labrador, Concepcion and Reyes
J.B.L., JJ., concur.
_________________________________________________________________________-G.R. No. L-58509 December 7, 1982
IN THE MATTER OF THE PETITION TO APPROVE THE WILL OF RICARDO B. BONILLA
deceased, MARCELA RODELAS, petitioner-appellant,
vs.
AMPARO ARANZA, ET AL., oppositors-appellees, ATTY. LORENZO SUMULONG, intervenor.
Luciano A. Joson for petitioner-appellant.
Cesar Paralejo for oppositor-appellee.

RELOVA, J.:
This case was certified to this Tribunal by the Court of Appeals for final determination pursuant to
Section 3, Rule 50 of the Rules of Court.
As found by the Court of Appeals:
... On January 11, 1977, appellant filed a petition with the Court of First Instance of Rizal
for the probate of the holographic will of Ricardo B. Bonilla and the issuance of letters
testamentary in her favor. The petition, docketed as Sp. Proc. No. 8432, was opposed by
the appellees Amparo Aranza Bonilla, Wilferine Bonilla Treyes Expedita Bonilla Frias
and Ephraim Bonilla on the following grounds:
(1) Appellant was estopped from claiming that the deceased left a will by failing to
produce the will within twenty days of the death of the testator as required by Rule 75,
section 2 of the Rules of Court;
(2) The alleged copy of the alleged holographic will did not contain a disposition of
property after death and was not intended to take effect after death, and therefore it was
not a will
(3) The alleged hollographic will itself,and not an alleged copy thereof, must be
produced, otherwise it would produce no effect, as held in Gam v. Yap, 104 Phil. 509;
and

(4 ) The deceased did not leave any will, holographic or otherwise, executed and attested
as required by law.
The appellees likewise moved for the consolidation of the case with another case Sp.
Proc. No, 8275). Their motion was granted by the court in an order dated April 4, 1977.
On November 13, 1978, following the consolidation of the cases, the appellees moved
again to dismiss the petition for the probate of the will. They argued that:
(1) The alleged holographic was not a last will but merely an instruction as to the
management and improvement of the schools and colleges founded by decedent Ricardo
B. Bonilla; and
(2) Lost or destroyed holographic wills cannot be proved by secondary evidence unlike
ordinary wills.
Upon opposition of the appellant, the motion to dismiss was denied by the court in its
order of February 23, 1979.
The appellees then filed a motion for reconsideration on the ground that the order was
contrary to law and settled pronouncements and rulings of the Supreme Court, to which
the appellant in turn filed an opposition. On July 23, 1979, the court set aside its order of
February 23, 1979 and dismissed the petition for the probate of the will of Ricardo B.
Bonilla. The court said:
... It is our considered opinion that once the original copy of the holographic will is lost, a
copy thereof cannot stand in lieu of the original.
In the case of Gam vs. Yap, 104 Phil. 509, 522, the Supreme Court held that 'in the matter
of holographic wills the law, it is reasonable to suppose, regards the document itself as
the material proof of authenticity of said wills.
MOREOVER, this Court notes that the alleged holographic will was executed on January
25, 1962 while Ricardo B. Bonilla died on May 13, 1976. In view of the lapse of more
than 14 years from the time of the execution of the will to the death of the decedent, the
fact that the original of the will could not be located shows to our mind that the decedent
had discarded before his death his allegedly missing Holographic Will.
Appellant's motion for reconsideration was denied. Hence, an appeal to the Court of Appeals in which it
is contended that the dismissal of appellant's petition is contrary to law and well-settled jurisprudence.
On July 7, 1980, appellees moved to forward the case to this Court on the ground that the appeal does not
involve question of fact and alleged that the trial court committed the following assigned errors:
I. THE LOWER COURT ERRED IN HOLDING THAT A LOST HOLOGRAPHIC
WILL MAY NOT BE PROVED BY A COPY THEREOF;
II. THE LOWER COURT ERRED IN HOLDING THAT THE DECEDENT HAS
DISCARDED BEFORE HIS DEATH THE MISSING HOLOGRAPHIC WILL;

III. THE LOWER COURT ERRED IN DISMISSING APPELLANT'S WILL.


The only question here is whether a holographic will which was lost or cannot be found can be proved by
means of a photostatic copy. Pursuant to Article 811 of the Civil Code, probate of holographic wills is the
allowance of the will by the court after its due execution has been proved. The probate may be
uncontested or not. If uncontested, at least one Identifying witness is required and, if no witness is
available, experts may be resorted to. If contested, at least three Identifying witnesses are required.
However, if the holographic will has been lost or destroyed and no other copy is available, the will can
not be probated because the best and only evidence is the handwriting of the testator in said will. It is
necessary that there be a comparison between sample handwritten statements of the testator and the
handwritten will. But, a photostatic copy or xerox copy of the holographic will may be allowed because
comparison can be made with the standard writings of the testator. In the case of Gam vs. Yap, 104 PHIL.
509, the Court ruled that "the execution and the contents of a lost or destroyed holographic will may not
be proved by the bare testimony of witnesses who have seen and/or read such will. The will itself must be
presented; otherwise, it shall produce no effect. The law regards the document itself as material proof of
authenticity." But, in Footnote 8 of said decision, it says that "Perhaps it may be proved by a photographic
or photostatic copy. Even a mimeographed or carbon copy; or by other similar means, if any, whereby the
authenticity of the handwriting of the deceased may be exhibited and tested before the probate court,"
Evidently, the photostatic or xerox copy of the lost or destroyed holographic will may be admitted
because then the authenticity of the handwriting of the deceased can be determined by the probate court.
WHEREFORE, the order of the lower court dated October 3, 1979, denying appellant's motion for
reconsideration dated August 9, 1979, of the Order dated July 23, 1979, dismissing her petition to
approve the will of the late Ricardo B. Bonilla, is hereby SET ASIDE.
SO ORDERED.
Teehankee, Actg. C.J., Melencio-Herrera, Plana, Vasquez and Gutierrez, Jr., JJ., concur.
G.R. No. L-32213 November 26, 1973
AGAPITA N. CRUZ, petitioner,
vs.
HON. JUDGE GUILLERMO P. VILLASOR, Presiding Judge of Branch I, Court of First Instance
of Cebu, and MANUEL B. LUGAY, respondents.
Paul G. Gorrez for petitioner.
Mario D. Ortiz for respondent Manuel B. Lugay.

ESGUERRA, J.:
Petition to review on certiorari the judgment of the Court First Instance of Cebu allowing the probate of
the last will a testament of the late Valente Z. Cruz. Petitioner-appellant Agapita N. Cruz, the surviving
spouse of the said decease opposed the allowance of the will (Exhibit "E"), alleging the will was executed
through fraud, deceit, misrepresentation and undue influence; that the said instrument was execute
without the testator having been fully informed of the content thereof, particularly as to what properties

he was disposing and that the supposed last will and testament was not executed in accordance with law.
Notwithstanding her objection, the Court allowed the probate of the said last will and testament Hence
this appeal by certiorari which was given due course.
The only question presented for determination, on which the decision of the case hinges, is whether the
supposed last will and testament of Valente Z. Cruz (Exhibit "E") was executed in accordance with law,
particularly Articles 805 and 806 of the new Civil Code, the first requiring at least three credible
witnesses to attest and subscribe to the will, and the second requiring the testator and the witnesses to
acknowledge the will before a notary public.
Of the three instrumental witnesses thereto, namely Deogracias T. Jamaloas Jr., Dr. Francisco Paares
and Atty. Angel H. Teves, Jr., one of them, the last named, is at the same time the Notary Public before
whom the will was supposed to have been acknowledged. Reduced to simpler terms, the question was
attested and subscribed by at least three credible witnesses in the presence of the testator and of each
other, considering that the three attesting witnesses must appear before the notary public to acknowledge
the same. As the third witness is the notary public himself, petitioner argues that the result is that only two
witnesses appeared before the notary public to acknowledge the will. On the other hand, private
respondent-appellee, Manuel B. Lugay, who is the supposed executor of the will, following the reasoning
of the trial court, maintains that there is substantial compliance with the legal requirement of having at
least three attesting witnesses even if the notary public acted as one of them, bolstering up his stand with
57 American Jurisprudence, p. 227 which, insofar as pertinent, reads as follows:
It is said that there are, practical reasons for upholding a will as against the purely
technical reason that one of the witnesses required by law signed as certifying to an
acknowledgment of the testator's signature under oath rather than as attesting the
execution of the instrument.
After weighing the merits of the conflicting claims of the parties, We are inclined to sustain that of the
appellant that the last will and testament in question was not executed in accordance with law. The notary
public before whom the will was acknowledged cannot be considered as the third instrumental witness
since he cannot acknowledge before himself his having signed the will. To acknowledge before means to
avow (Javellana v. Ledesma, 97 Phil. 258, 262; Castro v. Castro, 100 Phil. 239, 247); to own as genuine,
to assent, to admit; and "before" means in front or preceding in space or ahead of. (The New Webster
Encyclopedic Dictionary of the English Language, p. 72; Funk & Wagnalls New Standard Dictionary of
the English Language, p. 252; Webster's New International Dictionary 2d. p. 245.) Consequently, if the
third witness were the notary public himself, he would have to avow assent, or admit his having signed
the will in front of himself. This cannot be done because he cannot split his personality into two so that
one will appear before the other to acknowledge his participation in the making of the will. To permit
such a situation to obtain would be sanctioning a sheer absurdity.
Furthermore, the function of a notary public is, among others, to guard against any illegal or immoral
arrangement Balinon v. De Leon, 50 0. G. 583.) That function would defeated if the notary public were
one of the attesting instrumental witnesses. For them he would be interested sustaining the validity of the
will as it directly involves him and the validity of his own act. It would place him in inconsistent position
and the very purpose of acknowledgment, which is to minimize fraud (Report of Code Commission p.
106-107), would be thwarted.
Admittedly, there are American precedents holding that notary public may, in addition, act as a witness to
the executive of the document he has notarized. (Mahilum v. Court Appeals, 64 0. G. 4017; 17 SCRA
482; Sawyer v. Cox, 43 Ill. 130). There are others holding that his signing merely as notary in a will

nonetheless makes him a witness thereon (Ferguson v. Ferguson, 47 S. E. 2d. 346; In Re Douglas Will, N.
Y. S. 2d. 641; Ragsdal v. Hill, 269 S. W. 2d. 911, Tyson Utterback, 122 So. 496; In Re Baybee's Estate
160 N. 900; W. Merill v. Boal, 132 A. 721;See also Trenwith v. Smallwood, 15 So. 1030). But these
authorities do not serve the purpose of the law in this jurisdiction or are not decisive of the issue herein
because the notaries public and witnesses referred to aforecited cases merely acted as instrumental,
subscribing attesting witnesses, and not as acknowledging witnesses. He the notary public acted not only
as attesting witness but also acknowledging witness, a situation not envisaged by Article 805 of the Civil
Code which reads:
ART. 806. Every will must be acknowledged before a notary public by the testator and
the witnesses. The notary public shall not be required to retain a copy of the will or file
another with the office of the Clerk of Court. [Emphasis supplied]
To allow the notary public to act as third witness, or one the attesting and acknowledging witnesses,
would have the effect of having only two attesting witnesses to the will which would be in contravention
of the provisions of Article 80 be requiring at least three credible witnesses to act as such and of Article
806 which requires that the testator and the required number of witnesses must appear before the notary
public to acknowledge the will. The result would be, as has been said, that only two witnesses appeared
before the notary public for or that purpose. In the circumstances, the law would not be duly in observed.
FOR ALL THE FOREGOING, the judgment appealed from is hereby reversed and the probate of the last
will and testament of Valente Z. Cruz (Exhibit "E") is declared not valid and hereby set aside.
Cost against the appellee.
Makalintal, C.J., Castro, Teehankee, Makasiar and Muoz Palma, JJ., concur.

Das könnte Ihnen auch gefallen